Bar Prep - Torts

Ace your homework & exams now with Quizwiz!

Four different standards of care - premises of liability

(1) Unknown trespasser on property - somebody comes into the land without permission onto the property and owner is unaware a. There is NO duty of care owed to unknown trespasser on property b. Will never ever win a negligence claim! 2. Discovered / anticipated trespasser on property - someone the possessor sees and is currently on the land. Anticipated situation is usually one where there is a pattern of trespassing in the past, and owner of the property knows about it. a. Duty to protect in this situation, only comes about only in 4-part test: i. (1) Hazardous condition must be artificial in nature. 1. Opposite of this is natural occurring condition 2. No duty is owed to naturally occurring hazards on the land. Ex: dead branch of tree, icy snow and conditions, etc. ii. (2) Condition must be highly dangerous! 1. No duty is owed to only those conditions that are moderately dangerous iii. (3) Hazard must be concealed 1. Danger cannot be obvious, if it is, then you owe no duty. 2. In such a situation, entrant can protect himself iv. (4) Hazard must have been on that the defendant knew about in the past b. In other words, the property owner in this situation will only protect entrant from known, man-made, highly dangerous, death traps (concealed) on the land ß this is only duty 3. Licensees a. Express or implied i. Both à Do not confer economic benefit on possessor of the land b. Classic example is social guest (friend who comes to watch the game), or someone who comes and knocks on the door and there is no "no soliciting", sign, there is implied consent and they are licensees. (ex: girl scouts) c. Duty = only Two part test i. (1) Hazard must be concealed 1. Can't be open or obvious ii. (2) Hazard must have been on that the defendant knew about in the past 1. same as above, but deleted highly dangerous or artificial condition reqs iii. So only duty is duty to protect licensee from known, death traps on the land (any level of danger), in other words: duty to warn of nonobvious dangerous conditions that are known 4. Invitees a. Enter with permission and confer benefit onto possessor. Classic example: is patron of restaurant. i. Don't have to give money, ex: if you go to church or go to museum. b. Duty i. (1) same duty as licensees plus ii. (2) condition is one that possessor knew about in advanced or could have discovered through a reasonable inspection 1. Inspection that would have been taken by reasonably prudent person. 2. Only reasonable inspection is required c. Def must protect from all reasonably knowable traps on the land. d. Duty to warn of nonobvious dangerous conditions that are known and to make reasonable inspections to discover dangerous conditions and make them safe

When a statutory standard of care replaces the common law duty of care in a negligence case, the defendant's lack of compliance with the statute will be excused if:

(i) compliance with the statute would cause more danger than a violation (e.g., a defendant drives onto the wrong side of the road to avoid hitting children who dart into his path), or (ii) compliance with the statute would be beyond the defendant's control (e.g., a blind pedestrian crosses against a light).

What are some things to keep in mind when dealing with intentional torts?

* Legal capacity does not matter. 1) When you are deciding whether a plaintiff has a claim, the extreme or hypersensitivity of a person should be ignored - Ignore eccentricities - Assume you are dealing with a normal person with normal sensitivity 2) Legal incapacity does not matter. - Ex. if a child locks you in a closet, child is still liable for false imprisonment - NEVER pick choice in intentional torts question that says "defendant lacked legal capacity" 3) Transferred intent doctrine - If a Def intends to commit one tort, but a different tort happens, still liable for end tort

Trespass to Land

1) D intends to be where D is. 2) D commits an act of physical invasion 3) That act must interfere with P's exclusive possession of land. physical invasion element can be met by: (a) Go on someone else's property - defendant does not have to necessarily know that he is crossing on someone else's property! - Remember, it is still an intentional tort, but the intent is the intent to be on the land. He put one foot on the other, and intended to be there. a. Ex: Daniel trips and falls into property by accident, no intent so no tort (b) Deliberately throw physical object onto/through neighbor's land - Flashing light is not enough

Assault

1) D must place the P in reasonable apprehension that a battery is about to be committed 2) Apprehension must be of an immediate battery - apprehension = knowledge - you have to become aware that plaintiff is about to commit a battery, but don't have to be afraid - Remember: words alone are not enough to create immediacy, need words + conduct! - but sometimes words can negate immediacy, (ex: you lift your hands to slap but say if you weren't my friend I would slap you)

Intentional Infliction of Emotional Distress

1) Intent or recklessness (only intentional tort that doesn't require intent, can be satisfied with reckelessness) 2) An act by D must amount to extreme or outrageous conduct. 3) P must suffer from severe emotional distress. Outrageous-ness: i. Mere insults are [usually] never outrageous. ii. Scenarios that act as "plus" factors, that usually show outrageous conduct: (a) Where conduct in question is continuous or repetitive in nature. Ex: abusive debt collection practices (b) If defendant is common carrier or innkeeper: United airlines, Amtrak, hotels, etc. - So if they do something with intent to distress, then almost always outrageous (you're too ugly to stay here, hotel innkeeper says) (c) Plaintiff belongs to a fragile class: a. Children - ex: adult picking on a little child b. Elderly people c. Pregnant women (d) Defendant has advanced information that plaintiff has some kind of emotional weakness, then targeting that emotional weakness is itself outrageous Plaintiff's Injury - must suffer severe emotional distress i. 1) no specific evidence is required: - don't have to prove that you lost work, don't have to prove that you went to a doctor, or have to prove specific psychiatric examination, etc. - In theory, can just establish through direct examination, ii. (2) can't just be "mildly annoyed," look at answers to see if they were mildly annoyed - Momentarily upset, briefly irritated, etc.

Trespass to Chattels / Conversion

1) Intentional interference 2) with plaintiff's personal property if interference is slight --> trespass to chattels if interference is large --> conversion Personal property = chattels, i.e. anything you own except land and real estate Two ways to interfere with personal property: i. Damage it - scratch it, etc. ii. Taking it away from you - robbing you of it **A mistake about the ownership of the items won't reduce liability Ex: you think you were working on your laptop, get frustrated that its not working, and throw it across the room breaking it --> you acted intentionally at the moment and broke the laptop, even though you mistakenly thought it was yours --> liable conversion

False Imprisonment

1) The D must commit an act of physical restraint 2) P must be confined in a bounded area For first element: - Threats can be sufficient! (needs to be real and genuine) - Failure to act can be the requisite act of restraint, if there is some preexisting duty between these two people - an act of restraint only counts if the plaintiff is aware of it or harmed by it For second element: An area is not bounded if there is a reasonable means of escape that the plaintiff can reasonably discover i. On test: look to see if the way out is dangerous, disgusting, hidden, humiliating, etc. that is not a reasonable means of escape

What are the seven intentional torts?

1. Battery 2. Assault 3. False Imprisonment 4. Intentional Infliction of Emotional Distress 5. Trespass to Land 6. Trespass to Chattels 7. Conversion

List 6 Special Duty Topics

1. Children 2. Professionals 3. Premises Liability 4. Statutory Standards of Care 5. Duties to Act Affirmatively 6. Emotional Distress Claim

List elements of Negligence

1. Duty - the plaintiff must show that the defendant owed a duty and then specify the nature of that duty 2. Breach - the plaintiff must show that the defendant breached that duty 3. Causation - actual + proximate 4. Damage - injury, have to show harm

What are the three affirmative defenses to intentional torts?

1. consent 2. protective privileges 3. necessity Consent - ask if person has legal capacity to consent! a drunk can't consent to battery for example. two minor boys can consent to wrestle, etc. Express consent - a. an outright grant (oral or written) of permission to the defendant to behave in an outright way that otherwise would be considered a tort Implied consent - from custom and usage; ex: sports, body language, etc. Exceptions to either express or implied consent --> if consent was obtained through fraud or duress or failure to disclose, then the consent is void. ex: STD, failure to disclose during a hookup, then will be liable for battery because failed to disclose and there was physical contact that was offensive without proper consent Protective Privileges Types of protective privileges: 1. Self Defense 2. Defense or Property 3. Defense of Others a. Time - threat has to be in progress and imminent! i. Can't act too soon, or act too late, must be in the heat of the moment ii. No preemption and no revenge iii. Conflict not over until person disengages b. Accuracy - reasonable belief that the threat is genuine i. If defendant makes a reasonable mistake, the defendant still has this defense available ii. But remember that defense needs to be proportional (deadly force is always disproportional if all you are doing is defense of property) Necessity 1. Only apply to three property torts: trespass to land, conversion, or trespass to chattel 2. Types: a. Public necessity --> defendant commits a property tort in an emergency to protect the community as a whole or a significant group of people b. Private necessity --> defendant commits a property tort in an emergency but those so for personal reasons; catastrophe only affects him ß private necessity is only a partial defense. Three legal consequences: i. You have to pay for any actual harm you do, i.e. you remain liable for compensatory damages in a private necessity scenario ii. If you do no harm, your technical tort is excused, and therefore you will not be liable for nominal or punitive damages iii. If you go on someone's property during the emergency to take refuge, you must be allowed to remain. Property owner is not allowed to throw you off the land as long as the emergency continues. Ex: farmer joe can't kick hiker pete off his home if there is a blizzard outside once hiker pete enters (trespasses) into home for private necessity

Which of the following statements regarding proximate cause is true? (A) In direct cause cases, the unusual manner in which the injury occurred is not relevant. (B) In indirect cause cases, another force comes into play before the defendant's negligent act and combines with it to cause the injury. (C) A defendant may have proximately caused the plaintiff's injury even though she did not actually cause it. (D) Negligence of rescuers generally is not foreseeable.

A A direct cause case is one where the facts present an uninterrupted chain of events from the time of the defendant's negligent act to the time of plaintiff's injury. If a particular harmful result was at all foreseeable from the defendant's negligent conduct, the unusual timing of cause and effect or the unusual manner in which the injury occurred is not relevant to the defendant's liability. Negligence of rescuers is generally foreseeable because a rescue attempt is a common intervening force that is a normal response or reaction to the situation created by the defendant's negligent act (i.e., "danger invites rescue"). It is not true that a defendant may have proximately caused the plaintiff's injury even though she did not actually cause it. In fact, the opposite is true: A defendant may have actually caused the plaintiff's injury but not have proximately caused the injury, because proximate cause is a limitation on liability. In indirect cause cases, another force comes into play AFTER the defendant's negligent act and combines with it to cause the injur

If a statute providing for a criminal penalty is applicable to a common law negligence case, a clearly stated specific duty imposed by the statute will replace the more general common law duty of care. Most courts hold that violation of an applicable statute __________. (A) is negligence per se (B) raises a rebuttable presumption as to duty and breach (C) raises a rebuttable presumption as to breach (D) is only prima facie evidence of negligence

A Most courts still adhere to the rule that violation of an applicable statute is "negligence per se." This means that the plaintiff will have established a conclusive presumption of duty and breach of duty by showing a violation of the statute. (The plaintiff still must establish causation and damages to complete the prima facie case for negligence.) For a statute to apply in a negligence case: (i) The plaintiff must show that she is in the class intended to be protected by the statute; and (ii) The plaintiff must show that the statute was designed to prevent the type of harm that the plaintiff suffered. The minority view holds either that: (i) a rebuttable presumption as to duty and breach arises, or (ii) the statutory violation is only prima facie evidence of negligence.

To fight drug abuse, a state enacted a statute forbidding the selling of model airplane glue to anyone under the age of 18 except in small quantities in prepackaged model kits. Violation of the statute was penalized by fines or, in cases of multiple violations, possible imprisonment. The statute also required that all elementary and secondary schools licensed by the state provide comprehensive drug education programs. Neither the legislature nor the courts of the state have abolished the common law tort defense of assumption of the risk. The owner of a hobby shop in the state sold a large tube of airplane glue to a 15-year-old boy who reasonably appeared to be at least 18 years old. The boy had received drug education in his school, as mandated by the statute, including coverage of the dangers of glue sniffing. The boy understood the anti-drug instruction, but he wanted to experience it for himself. The boy sniffed the glue repeatedly and suffered permanent brain damage. If the boy's parents file suit on the boy's behalf against the store owner, for whom is the court likely to rule? (A) For the boy, because the store owner violated the statute when she sold the glue to the boy. (B) For the store owner, because the boy is not a member of the class of persons meant to be protected by the statute. (C) For the store owner, because the boy was aware of the danger when he sniffed the glue. (D) For the store owner, because the boy could reasonably have been mistaken for an 18-year-old by the store owner.

A The boy will prevail because the store owner's violation of the statute resulted in the boy's injuries. The applicable standard of care in a cause of action can be established by proving the applicability to that action of a statute providing for a criminal penalty. If this is done, a clearly stated specific duty imposed by the statute will replace the more general common law duty of care. For the statutory standard to be applicable, the plaintiff must show that (i) he is within the class intended to be protected by the statute, and (ii) the statute was designed to prevent the type of harm that the plaintiff suffered. Here, the statute clearly prohibited the transaction that took place between the boy and the store owner. It establishes the standard of care because the boy is a minor purchasing a large quantity of model airplane glue, and the serious injury he suffered from sniffing the glue was one of the harms that the statute was designed to prevent. The effect of establishing a violation of the statute is that a conclusive presumption of duty and breach of duty is established. The boy should then be able to establish that the store owner's sale of the glue was the actual cause and proximate cause of the boy's injuries, completing the prima facie case of negligence on the store owner's part. (B) is incorrect because minors such as the boy who were experimenting with glue sniffing were a primary target of the statute. (C) is incorrect because the boy's assumption of the risk is not a defense under these circumstances. The facts indicate that the state has retained the common law tort defense of assumption of the risk. Under this defense, a plaintiff will be denied recovery in a negligence action if he either expressly or impliedly knew of the risk of injury and voluntarily proceeded in the face of the risk. However, courts refuse to permit an assumption of risk defense in some situations because of public policy considerations. When a statute applies and is enacted to protect a class, members of that class will not be deemed to have assumed any risk. Here, even though the boy was aware of the danger when he voluntarily sniffed the glue, the statute was enacted to protect minors such as the boy from the dangers of glue sniffing. Thus, the store owner cannot rely on assumption of risk as a defense. (D) is incorrect because the statute does not provide for a reasonable mistake to excuse its violation. If the common law duty of reasonable care were applicable here, the reasonableness of the store owner's mistake would be relevant. However, the statute's specific duty replaces the more general duty of reasonable care, and violation of a statutory standard will only be excused where compliance would cause more danger than violation or would be beyond the defendant's control. Neither situation is indicated here, so the statute applies.

A petroleum company operated refineries in several states and was also engaged in the manufacture of a variety of petrochemical products. The company hired an industrial cleaning service to thoroughly clean one of its refineries. While one of the cleaning service's employees was engaged in routine cleaning activities at the refinery, one of the support legs on a crane suddenly gave way, causing part of the crane to fall onto a pipe carrying hot oil, cracking it open. The employee had his back to the pipe at the time and hot oil squirted over his back and legs, causing severe burns. The employee filed suit against the petroleum company for his injuries. The parties stipulated for trial that the crane had been designed and constructed by a crane construction specialist and was serviced at regular intervals by a reputable crane maintenance company selected by the crane construction company. The employee testified at the trial that he was injured when the pipe cracked open and submitted his medical bills and other evidence of damages. The employee introduced no further evidence. At the conclusion of the employee's case, the petroleum company moved for a directed verdict in its favor. Should the directed verdict be granted? (A) Yes, because the employee has done nothing to connect the petroleum company to any negligent activity that might have caused the accident. (B) Yes, because the petroleum company did not owe a duty to an employee of an independent contractor. (C) No, because the petroleum company is strictly liable to the employee for his injuries. (D) No, because a jury could reasonably conclude, based on the evidence presented by the employee, that the petroleum company was negligent.

A The court should grant the petroleum company's motion for a directed verdict in its favor because the employee has not established a prima facie case against the petroleum company. The question does not indicate the theory of liability for the employee's lawsuit; however, because strict liability is not applicable against the petroleum company for operation of the refinery (as discussed below) and because there is no evidence to establish that the petroleum company is vicariously liable for another party's negligence here, the employee's only feasible theory of liability is that the petroleum company itself was negligent. While the employee has established the negligence elements of duty, causation, and damages, he has not established the element of breach of duty. While breach of duty is ordinarily a question for the trier of fact, plaintiff's failure to offer any evidence on that element of the prima facie case will permit a directed verdict for defendant. Under certain circumstances, the fact that a particular injury occurred may itself establish or tend to establish a breach of duty owed, permitting the trier of fact to infer defendant's liability. This is the doctrine of res ipsa loquitur. However, for the doctrine to apply, plaintiff must show that: (i) the accident causing his injury is the type that would not normally occur unless someone was negligent; (ii) the negligence was attributable to defendant; and (iii) the injury was not attributable to plaintiff. For the second requirement, plaintiff must establish that this type of accident ordinarily happens because of the negligence of someone in defendant's position. This can often be done by showing that the instrumentality causing the injury was in the exclusive control of the defendant. Here, however, the crane that caused the injury was designed and constructed by a company other than the petroleum company and was serviced and maintained by still another company not selected by the petroleum company. Even assuming that the collapse of the crane was the type of accident that does not normally occur unless someone was negligent, there is no evidence that the petroleum company was the source of that negligence. The accident could well be attributable to negligence on the part of the manufacturer or the company hired by the manufacturer to service the crane, or simply to a defect in the materials used to construct the crane, and there is no basis for the petroleum company's being vicariously liable for the actions of either company, since the exceptions that impose vicarious liability for the conduct of an independent contractor do not apply. Since no other evidence of breach of duty was established, the petroleum company's motion for a directed verdict should be granted. (B) is incorrect because the petroleum company owed a duty to the employee since the employee was an invitee on the petroleum company's property. An invitee is one who enters onto the premises in response to an express or implied invitation of the landowner or occupier, including those who enter for a purpose connected with the business interests of the landowner. Here, even though the employee was an employee of an independent contractor, he was on the premises for the benefit of the petroleum company's refinery operations and at its invitation. Thus, he is an invitee to whom the petroleum company owed a duty of reasonable care. (C) is incorrect because the refinery operation is not an abnormally dangerous activity. For strict liability to apply to an activity, the activity (i) must create a foreseeable risk of serious harm even when reasonable care is exercised by all actors, and (ii) must not be a matter of common usage in the community. Because an oil refinery can be operated in many locations without the risk of serious harm as long as due care is exercised, a court is not likely to find it to be an abnormally dangerous activity; hence, the petroleum company would not be strictly liable to the employee. (D) is incorrect because, as discussed above, the employee has presented no evidence of the petroleum company's negligence and has therefore failed to establish his prima facie case.

Which of the following statements of law does NOT relate to proving actual cause in a strict products liability case? (A) The defendant cannot avoid liability by showing negligent failure of an intermediary to discover the defect. (B) If the defect is difficult to trace, the plaintiff may rely on an inference that such a product failure ordinarily would occur only as a result of a defect. (C) If the defect has inadequate warnings, the plaintiff is entitled to a presumption that an adequate warning would have been read and heeded. (D) The defect in the product must have existed when the product left the defendant's control.

A The rule that the defendant cannot avoid liability by showing negligent failure of an intermediary to discover the defect does not relate to actual cause; rather, it relates to the proximate cause principle that negligence of an intermediary is foreseeable and not a superseding cause. Under this principle, the intermediary's negligence does not cut off the defendant's liability for supplying a defective product. The basic requirement to show actual cause is that the defect in the product must have existed when the product left the defendant's control. When a defect is difficult to establish (such as if the product is destroyed), the plaintiff may rely on an inference that such a product failure ordinarily would occur only as a result of a defect (similar to res ipsa loquitur). To show that inadequate warnings were an actual cause of the injury, the plaintiff is entitled to a presumption that an adequate warning would have been read and heeded (i.e., but for the lack of an adequate warning, the plaintiff would not have been injured).

A shopper at a grocery store slipped and fell when he stepped in some water that had seeped out from a malfunctioning freezer case. The fall caused the shopper to break an ankle, so he filed suit against the store in a jurisdiction applying the traditional rules for landowners and possessors of land. At trial, the shopper presented evidence of the above facts, and testified that the floor around the water appeared dirty. To survive a motion for summary judgment by the store, what additional evidence must the shopper present? (A) No additional evidence. (B) He was planning to make a purchase at the store. (C) The store employees knew that the freezer case was leaking. (D) His attention was diverted by store displays so that he did not notice the water on the floor.

A The shopper's lawsuit will survive a motion for summary judgment by the store without any additional evidence. Under the facts here, the shopper was an invitee as to the store because he came onto the premises for a purpose connected with the store's business. The store therefore owed him the duty to warn of nonobvious dangerous conditions and to make reasonable inspections to discover dangerous conditions and make them safe. The shopper's testimony that the floor around the water appeared dirty suggests that the floor had not been swept or mopped for some time. This is enough evidence to allow the jury to decide whether the store employees failed to reasonably inspect or make safe an area in which its invitees would walk, which would breach its duty to the shopper. (B) is incorrect because the shopper need not show that he planned to make a purchase to have the status of an invitee. Even if he came just to return an item or browse the aisles and compare prices, he qualifies as an invitee. (C) is incorrect because the store could be liable even if its employees did not know that water was leaking onto the floor. Because the shopper was an invitee, the store owed a duty to make reasonable inspections to discover unsafe conditions. (D) is incorrect because the shopper need not establish his due care here. Even if the shopper was not distracted by displays and should have seen the water had he been watching where he was walking, he can still recover some damages under pure comparative negligence, which allows recovery against a negligent defendant no matter how great plaintiff's negligence is. It will be an issue for the jury to determine whether and to what extent the shopper was at fault.

A 16-year-old teenager was playing baseball in a sandlot when the ball was hit over his head and onto a landowner's adjacent property. Ignoring "beware of dog" signs, the teenager climbed over the fence into the landowner's yard to retrieve the ball and was attacked by a vicious guard dog belonging to the landowner. The dog bit the teenager, causing him to suffer severe lacerations that required numerous stitches. If the teenager brings an action against the landowner to recover damages for his injuries, will he likely prevail? (A) Yes, because the landowner may not use a vicious dog to protect only his property. (B) Yes, because the landowner is strictly liable for injuries caused by the vicious dog. (C) No, because the teenager was trespassing on the landowner's property. (D) No, because the landowner had posted signs warning about the dog.

A The teenager will prevail because the landowner may not intentionally use a vicious dog to protect only his property. One may use only reasonable force to defend property. A landowner may not use force that will cause death or serious bodily harm. Furthermore, one may not use indirect deadly force such as a trap, spring gun, or vicious dog when such force could not lawfully be directly used, e.g., against a mere trespasser. (B) is incorrect because strict liability in such cases generally is not imposed in favor of undiscovered trespassers against landowners. Trespassers cannot recover for injuries inflicted by the landowner's abnormally dangerous domestic animals in the absence of negligence. (C) is incorrect because a landowner who protects his property from intruders by keeping a vicious watchdog he knows is likely to cause serious bodily harm may be liable even to trespassers for injuries caused by the animal. The liability is based on intentional tort principles: Because the landowner is not entitled to use deadly force in person to protect only property, he also may not use such force indirectly. (D) is incorrect because even though the landowner posted warning signs, he can still be liable under intentional tort principles because he intentionally used the dog to protect his property, knowing that the dog is likely to cause serious bodily harm.

A principal will be vicariously liable for the tortious acts of her independent contractor __________.

A principal will be vicariously liable for the tortious acts of her independent contractor if the independent contractor is engaged in inherently dangerous activities. In general, a principal will not be vicariously liable for tortious acts of an independent contractor. Two broad exceptions exist, however: (i) The independent contractor is engaged in inherently dangerous activities, e.g., excavating next to a public sidewalk, blasting; or (ii) The duty, because of public policy considerations, is simply nondelegable, e.g., the duty of a business to keep its premises safe for customers. Respondeat superior is the doctrine that makes employers vicariously liable for the torts of employees; it does not apply to independent contractors. A principal may be liable for negligently selecting or supervising an independent contractor. However, that liability is for her own negligence; it is not vicarious liability.

Which of the following persons is considered to be an invitee of the landowner? (A) A hiker hiking on the landowner's open land with permission. (B) A child accompanying a customer of the landowner. (C) A firefighter fighting a fire on the landowner's property. (D) A customer of the landowner who goes through a door marked "employees only."

B A child accompanying a customer of the landowner is considered an invitee because she came onto the property for a purpose connected to the business. Under traditional landowner liability rules, a landowner owes an invitee a general duty to use reasonable and ordinary care in keeping the property reasonably safe for the benefit of the invitee. This general duty includes the duties owed to licensees (to warn of nonobvious, dangerous conditions known to the landowner and to use ordinary care in active operations on the property). A landowner also owes invitees a duty to make reasonable inspections to discover dangerous conditions and make them safe. A customer of the landowner who goes through a door marked "employees only" is no longer an invitee. A person loses his status as an invitee if he exceeds the scope of the invitation-if he goes into a portion of the premises where his invitation cannot reasonably be said to extend. A firefighter fighting a fire on the landowner's property is not treated like an invitee. Under the "firefighter's rule," police officers and firefighters are generally treated like licensees, based on public policy or assumption of risk grounds. They cannot recover for a landowner's failure to inspect or repair dangerous conditions that are an inherent risk of their law enforcement or firefighting activity. A hiker on the landowner's open land is not considered an invitee. If an owner or occupier of open land permits the public to use the land for recreational purposes without charging a fee, the landowner is not liable for injuries suffered by a recreational user unless the landowner willfully and maliciously failed to guard against or warn of a dangerous condition or activity.

Which of the following is correct regarding strict liability for abnormally dangerous activities? (A) To be characterized as an abnormally dangerous activity, the activity must be considered abnormally dangerous in every community. (B) To be abnormally dangerous, the activity must create a foreseeable risk of serious harm even when reasonable care is exercised by all actors. (C) Strict liability will arise for any type of harm caused by engaging in the abnormally dangerous activity. (D) Whether an activity is abnormally dangerous is a question of fact for the jury to decide.

B An activity may be characterized as abnormally dangerous if it creates a foreseeable risk of serious harm even when reasonable care is exercised by all actors. Determining whether an activity is abnormally dangerous is NOT a question of fact for the jury to decide; rather, it is a question of law that the court can decide on a motion for a directed verdict. It is incorrect that strict liability will arise from any type of harm caused by the activity. The harm must result from the kind of danger to be anticipated from the abnormally dangerous activity; i.e., it must flow from the "normally dangerous propensity" of the condition or thing involved. It is also incorrect that the activity must be considered abnormally dangerous in every community. Courts generally impose a requirement that the activity must not be a matter of common usage in the community where it takes place. An activity may be considered abnormally dangerous in some areas but not in others.

A landlord employed his friend as the on-site manager of one of his apartment buildings despite being aware that he had previously been arrested for criminal battery, disorderly conduct, and driving while intoxicated. The manager did a good job dealing with the general maintenance of the apartment building, although the landlord was aware that he continued to drink heavily. One night the manager, who was extremely intoxicated, attempted to swat an insect on the ceiling of his apartment and could not do so after several attempts. Enraged, he took a pistol from his drawer and shot at the insect. The bullet missed the insect and passed through the ceiling of his apartment into the apartment above, lodging in the leg of a tenant's social guest. Does the guest have a viable cause of action against the landlord? (A) Yes, because the guest had been invited onto the property by the tenant. (B) Yes, because the landlord was aware of the manager's habitual drunkenness and propensity for violence. (C) No, because the landlord cannot be held liable for the manager's intentional torts. (D) No, because shooting an insect was outside the scope of the manager's employment.

B Because the landlord knew about the manager's continued heavy drinking and tendencies toward violence, the guest has a cause of action for negligence in the landlord's hiring of the manager. An employer owes a duty to all those who may foreseeably come into contact with his employee to exercise due care in the hiring, supervision, and retention of the employee, and the landlord's retention of the manager under these circumstances may be a breach of that duty. (A) is incorrect because the landlord's liability here is based on negligent hiring rather than the guest's status on the property. (C) is also incorrect. An employer can be held directly liable for the intentional tort of an employee if it was foreseeable and the employer was negligent in hiring or retaining the employee. (D) is a true statement that would be relevant for vicarious liability purposes. However, it does not preclude the landlord from being liable for his own negligence based on the foreseeability of his employee acting violently.

A bottler markets water in lightweight plastic bottles that are sold by grocery stores, sporting goods stores, and other retail outlets. A hiker purchased several bottles of the water from a retailer and took them with him on a hike. While the hiker left his backpack unattended, a thief took one of the unopened containers without permission and drank some of the water. He immediately became violently ill. Tests were run on the water and showed that it contained impurities. If the thief maintains a negligence action against the bottler, which of the following arguments would be the most helpful to the bottler in avoiding liability? (A) The retailer had ample opportunity to test and inspect samples of the bottled water for purity and failed to do so. (B) The bottler bottled its water in compliance with numerous statutes that regulate the process of bottling water for human consumption. (C) The thief has failed to introduce any evidence at trial as to how the impurities got into the water he drank, and therefore has not met his burden of proof. (D) No reasonable person would have foreseen that the water would have been stolen and consumed by a thief.

B Evidence that the bottler complied with applicable statutes will be admissible to show that the bottler acted with ordinary, reasonable care, and is the only one of the listed arguments that would be helpful to the bottler. The bottler is being sued on a negligence theory; thus, the thief must prove that the bottler failed to exercise ordinary, reasonable care in bottling and distributing the water. Violation of a statute will establish a conclusive presumption of duty and breach of duty. However, compliance with an applicable statute does not necessarily establish due care, because due care may require more than is called for by the statute. Nevertheless, compliance with a statute is admissible as evidence that a defendant may have acted with due care. Thus, the bottler could use its compliance with the water bottling statutes as a means of establishing that it conformed with its duty to use ordinary, reasonable care. (A) is incorrect because a products liability action based on negligence uses the same causation analysis as a standard negligence case. Thus, a defendant's liability is not cut off by a foreseeable intervening force that comes into motion after the defendant's original negligent act. Consequently, an intermediary's negligent failure to discover a defect is not a superseding cause, and the defendant whose original negligence created the defect will be held liable along with the intermediary. Hence, the retailer's possibly negligent failure to inspect the water for purity will not relieve the bottler of liability for the consequences of its own negligence, if any. (C) will not be helpful to the bottler because this question allows for use of res ipsa loquitur. Under this doctrine, if a plaintiff shows that his injury is of a type that would not normally occur in the absence of negligence, and that such negligence is attributable to the defendant (e.g., by showing that the instrumentality causing the injury was in the exclusive control of the defendant), the trier of fact is permitted to infer the defendant's negligence. Here, impurities would not normally get into the bottled water in the absence of negligence, and the fact that the container from which the thief drank was unopened allows the trier of fact to infer that the impurity entered the water due to negligence on the part of the bottler. Therefore, the thief is not required to introduce evidence as to how the impurity got into the water in order to prevail. (D) is incorrect because the bottler's duty of due care in the context of products liability arises from having placed the water into the stream of commerce. Having done so, the bottler owes a duty to any foreseeable plaintiff, whether such person be an actual purchaser of the water or merely a user thereof. With the placing of the water into the stream of commerce, the thief is a foreseeable plaintiff as a drinker of the water, regardless of the fact that he obtained the water by means of theft.

A worker at a petrochemical plant was severely burned when a pipe carrying hot oil exploded. The worker brought a negligence action against the company that manufactured and installed the pipe. At trial, the worker established what happened and the injuries he suffered. He also presented evidence that the pipe burst because it had corroded at a higher than normal rate, which according to testimony of the worker's experts indicated a defect in the manufacture of the pipe. At the close of the worker's case, the manufacturer moved for a directed verdict. How should the court rule? (A) Deny the motion, because the pipe was defective and injured the worker. (B) Deny the motion, because the jury could find that the premature corrosion of the pipe would not have occurred absent negligence by the manufacturer. (C) Grant the motion, because the worker has not established that the manufacturer was negligent. (D) Grant the motion, because the pipe was in the petrochemical plant's possession when it exploded.

B The court should deny the motion because the jury may draw an inference of negligence from the plaintiff's evidence. The plaintiff's action against the manufacturer is a products liability action based on a negligence theory. In such a case, the prima facie case consists of: (i) a legal duty owed by the defendant to this plaintiff; (ii) breach of the duty; (iii) actual and proximate cause; and (iv) damages. Breach of duty requires showing (i) negligent conduct by the defendant leading to (ii) the supplying of a defective product by the defendant. The plaintiff may invoke res ipsa loquitur against the manufacturer if the error is something that usually does not occur without the negligence of the manufacturer. Here, the plaintiff has presented evidence that the manufacturer supplied a pipe that was so defective as to be unreasonably dangerous (because of its premature corrosion). The plaintiff can use res ipsa loquitur to show negligence because the manufacturer fabricated and installed the pipe and the premature corrosion would not likely have occurred without negligence on its part. Because the plaintiff has presented evidence of the other elements of the prima facie case, it should withstand the defendant's motion for directed verdict. (A) is incorrect because it implies liability without fault. As a plaintiff in a negligence action, the plaintiff must show that the manufacturer breached a duty owed to him, and that such breach caused his injuries. The mere fact that a pipe manufactured by the manufacturer exploded does not satisfy this burden. It is possible that the pipe could have exploded without any fault on the part of the manufacturer. (C) is incorrect because, as discussed above, the worker may rely on res ipsa loquitur here to establish an inference of negligence. (D) is incorrect because, despite the fact that the pipe was in the petrochemical plant's possession at the time of the explosion, the explosion itself may have been caused by negligence on the part of the manufacturer. Because the manufacturer fabricated and installed the pipe, the plaintiff may rely on res ipsa loquitur even though the manufacturer was not in possession of the pipe when the explosion occurred.

Which of the following is correct regarding proximate cause in negligence? (A) Proximate cause is required only when an intervening force contributes to the injury. (B) A defendant is liable for all harmful results caused by his acts. (C) A defendant is not liable for the harmful results of his conduct that are unforeseeable. (D) Proximate cause is not required for a defendant to be liable, but actual cause is required.

C A defendant is not liable when the harmful results of his conduct are unforeseeable. If a defendant's negligent conduct creates a risk of a harmful result, but an entirely different and totally unforeseeable type of harmful result occurs, most courts hold that the defendant is not liable for that harm. Proximate cause is required in a negligence action. In addition to being a cause in fact, the defendant's conduct must also be a proximate cause of the injury. The doctrine of proximate causation is a limitation of liability and deals with liability or nonliability for unforeseeable or unusual consequences of one's acts. A defendant is NOT liable for all harmful results caused by his acts. Not all injuries actually caused by the defendant will be deemed to have been proximately caused by his acts. The general rule of proximate cause is that the defendant is liable for only the harmful results that are the normal incidents of, and within the increased risk caused by, his acts (i.e., foreseeable results). Proximate cause is NOT required only when an intervening force contributes to the injury (indirect cause cases). Even in direct cause cases, the proximate cause doctrine may limit liability in a rare case.

Force may not be used by __________. (A) a property owner to defend property from tortious interference (B) a citizen in effecting a misdemeanor arrest (C) a landowner to regain real property after being tortiously dispossessed (D) an owner of chattel to recapture the chattel

C A landowner may not use force to regain real property after being tortiously dispossessed. Most states today do not allow resort to "self-help"; one who has been wrongfully excluded from possession of real property may bring an ejectment action or other summary procedure to recover possession. Hence, the owner who uses force to retake possession is liable for whatever injury she inflicts. (In former years, under the common law, a landowner tortiously dispossessed of real property could use reasonable force to regain possession, if she acted promptly upon discovery of the dispossession.) An owner of chattel may use force to recapture the chattel. An owner may use reasonable force to recapture a chattel when in "hot pursuit" of the tortfeasor. A demand for return of the chattel must be made before force is used, unless the demand would be futile or dangerous. However, force can be used only against the tortfeasor or a third party who knows that the chattel was tortiously obtained. If an innocent third party has obtained the chattel, the owner is no longer privileged to use force to effect a recapture of the chattel. A citizen may use force to effect a misdemeanor arrest. However, the citizen is allowed to use only the amount of force necessary to effect the arrest and never deadly force. A property owner may use force to defend the property from tortious interference. Although a property owner may use reasonable force to defend property, she may not use force that will cause death or serious bodily harm. Furthermore, one may not use indirect deadly force such as a trap, spring gun, or vicious dog when such force could not lawfully be directly used, e.g., against a mere trespasser.

Which of the following is true of the duty owed to a licensee by a landowner? (A) The landowner must repair known dangerous conditions on the land of which the licensee is not aware. (B) The landowner owes no duty to protect the licensee from active operations on the land. (C) The landowner owes a duty to warn of or make safe known dangerous conditions on the land of which the licensee is not aware. (D) The landowner must inspect for dangerous conditions on the land.

C A landowner owes a duty to a licensee to warn of or make safe known dangerous conditions on the land of which the licensee is not aware. A licensee is one who enters on land with the landowner's permission, express or implied, for her own purpose or business, rather than for the landowner's benefit. The owner has a duty to warn of or make safe a dangerous condition known to the owner that creates an unreasonable risk of harm to the licensee and that the licensee is unlikely to discover. As to a licensee, the landowner does NOT need to inspect for dangerous conditions on the land. Similarly, the landowner need NOT repair known dangerous conditions on the land of which the licensee is not aware; a warning generally will suffice. The landowner DOES have a duty to protect the licensee from active operations on the land. The owner has a duty to exercise reasonable care in the conduct of active operations for the protection of a licensee whom he knows to be on the property.

Which of the following is accurate regarding the defense of property? (A) landowner may use deadly force to defend her property. (B) A landowner cannot make a mistake as to whether an intrusion of her property has occurred. (C) A landowner usually must make a request to desist before defending her property. (D) A landowner's right to defend her property supersedes other privileges.

C A landowner usually must make a request to desist before defending her property. A request is not required if the circumstances make it clear that the request would be futile or dangerous. A landowner's right to defend her property does NOT supersede other privileges. If another is privileged to enter upon a landowner's property, due to necessity, a right of reentry, right to recapture chattels, etc., that privilege supersedes the landowner's right to defend her property. A landowner CAN make a mistake as to whether an intrusion of her property has occurred. A reasonable mistake is allowed as to the landowner's right to use force in defense of property, if the mistake involves whether an intrusion has occurred or whether a request to desist is required. A mistake is not allowed, however, if the entrant has a privilege to enter the property that supersedes the landowner's right to defend her property. Then the landowner is liable for her mistake, unless the entrant intentionally or negligently caused the mistake. A landowner may NOT use deadly force to defend her property. One can only use reasonable force to defend her property, not force that will cause death or serious bodily harm. Deadly force can be employed only if the landowner or another on the property is physically threatened, such that she may act in self-defense or defense of others.

Which of the following is not required for a statutory standard of care to apply in a negligence case? (A) The statute must have been designed to prevent the type of harm that occurred (B) The statute must state clearly what standard of conduct is expected (C) The statute must provide for a civil penalty (D) The plaintiff must be in the class intended to be protected by the statute

C The standard of care in a common law negligence case may be established by proving the applicability of a statute that provides for a criminal penalty, NOT a civil penalty. If the statute applies, the statute's specific duty will replace the more general common law duty of due care. If the statute in question provides for a civil remedy, the plaintiff will sue directly under the statute: i.e., it is not a common law negligence case. For a statute to apply, the specific duty imposed by the statute must be clearly stated. The plaintiff also must show that: (i) She is in the class intended to be protected by the statute; and (ii) The statute was designed to prevent the type of harm that the plaintiff suffered.

When a statutory standard of care replaces the common law duty of care in a negligence case, the defendant's lack of compliance with the statute will be excused if: (A) The defendant otherwise exercised due care (B) Violation of the statute causes more danger than compliance (C) Compliance with the statute is beyond the defendant's control (D) Violation of the statute would result in a civil infraction instead of a criminal one

C A violation of an applicable statute may be excused if: (i) compliance with the statute would cause more danger than a violation (e.g., a defendant drives onto the wrong side of the road to avoid hitting children who dart into his path), or (ii) compliance with the statute would be beyond the defendant's control (e.g., a blind pedestrian crosses against a light). Violation of the statute will not be excused if violation of the statute causes more danger than compliance; this misstates the rule stated above. It is not correct to state that violation of the statute will be excused if violation of the statute would result in a civil infraction instead of a criminal one; rather, the statute itself supplies the remedy rather than damages through a common law negligence action. The defendant will not be excused for violating the statute. Violation of the statute will not be excused if the defendant otherwise exercised due care. When a statute is applicable, the ordinary standard of due care is replaced by the standard of care established by the statute.

For the doctrine of res ipsa loquitur to apply, the plaintiff must establish that _________. (A) The defendant's breach of duty was the sole cause of the plaintiff's injury (B) The defendant possessed the instrumentality that caused the injury (C) The accident would not normally occur unless someone was negligent (D) The defendant violated a statute establishing a standard of care

C For the doctrine of res ipsa loquitur to apply, the plaintiff must establish that the accident causing his injury is the type that would not normally occur unless someone was negligent. The circumstantial evidence doctrine of res ipsa loquitur deals with situations where the fact that a particular injury occurred tends to establish a breach of a duty owed. Res ipsa loquitur requires that the plaintiff present evidence connecting the defendant with the negligence that occurred in order to support a finding of liability. This requirement can be satisfied by showing that the instrumentality that caused the injury was in the exclusive control of the defendant, but actual possession of the instrumentality is NOT necessary. It is not necessary to show that the defendant violated a statute establishing a standard of care. Establishing negligence by application of res ipsa loquitur is distinct from establishing negligence through the violation of a statute. The doctrine of res ipsa loquitur is a means of establishing breach of duty; it does not require a showing that the defendant's conduct was the sole cause of the plaintiff's injury.

Which of the following situations involves a common intervening force that courts almost always find foreseeable? (A) A roofer negligently leaves a hammer on the plaintiff's roof, and a strong wind blows the hammer off the roof, where it strikes the plaintiff (B) A parking lot attendant negligently leaves the keys to the plaintiff's car inside it with the doors unlocked, and a thief steals the car (C) A defendant negligently causes a plaintiff to break her leg, and while walking on her crutches, the plaintiff loses her balance and injures herself (D) A defendant negligently blocks a sidewalk, forcing the plaintiff to walk in the roadway, where he is struck by a negligently driven car

C If the plaintiff loses her balance while on crutches and injures herself, this involves an intervening force that is a normal response or reaction to the situation created by the defendant's act. This is a subsequent accident situation, where the plaintiff suffers a subsequent injury following her original injury, and the original injury was a substantial factor in causing the second accident. This is a type of intervening force that courts almost always find foreseeable. The other situations involve intervening forces that operate on a situation created by a defendant's negligence but are independent actions, rather than natural responses or reactions to the situation. An independent negligent act of a third person, a criminal act of a third person, and an act of God are all independent intervening forces. While these may be found to be foreseeable if the defendant's negligence created a significant risk that these forces would cause harm to the plaintiff, other types of intervening forces are deemed foreseeable because they are normal responses or reactions to the situation created by the defendant's act.

The "shopkeeper's privilege" allows a shopkeeper to avoid liability for false imprisonment when detaining a suspect that he reasonably believes has committed a theft. The shopkeeper also must: (A) Conduct the detention in a reasonable manner, detain the suspect for only a reasonable time, and notify the police in a reasonable amount of time. (B) Detain the suspect for only a reasonable time and notify the police in a reasonable amount of time. (C) Conduct the detention in a reasonable manner and detain the suspect for only a reasonable time. (D) Conduct the detention in a reasonable manner and notify the police in a reasonable amount of time.

C In addition to having a reasonable belief as to the fact of theft, a shopkeeper is required to conduct the detention in a reasonable manner and detain the suspect for a reasonable period of time for the privilege to apply. By statute in some states and case law in others, shopkeepers have been given a privilege to detain someone suspected of shoplifting and thus avoid liability for false imprisonment. The following conditions must be satisfied: 1. There must be a reasonable belief as to the fact of theft; 2. The detention must be conducted in a reasonable manner and only nondeadly force can be used; and 3. The detention must be only for a reasonable period of time and only for the purpose of making an investigation. A shopkeeper is not required to notify the police in a reasonable amount of time to avoid liability for false imprisonment when detaining a suspect for shoplifting

What is one of the effects of res ipsa loquitur? (A) It requires the defendant to present evidence of due care in rebuttal. (B) The burden of proof switches to the defendant. (C) No directed verdict may be given for the defendant. (D) It creates a presumption of negligence.

C One of the effects of res ipsa loquitur is that no directed verdict may be given for the defendant, because when the res ipsa element has been proved, the plaintiff has made a prima facie case for negligence. The doctrine, however, does NOT switch the burden of proof to the defendant and does NOT create a presumption of negligence. Furthermore, it does NOT require the defendant to present evidence of due care in rebuttal. If the jury elects not to infer negligence, it may find for the defendant even if the defendant presents no evidence on that issue.

A fire broke out in a home that had been recently remodeled, destroying the house and injuring the homeowner. An investigation by the fire marshal established that the fire started from a short in some wiring behind a wall. A small section of wiring that ran to an outlet through a narrow gap between a furnace chimney and a hot water pipe had had part of its outer sheath cut off. The homeowner filed suit against the electrical company that did the rough wiring. The parties stipulated for trial that the company had installed the wiring in compliance with the blueprints, and that the wiring had been inspected and approved by the building inspector before the chimney and the water pipe had been installed and the walls put up, all by different contractors. At trial, the homeowner introduced the report of the fire marshal establishing how the fire started, and evidence of his medical expenses and other damages. At the end of the homeowner's case, the electrical company's attorney rested her case and moved for a directed verdict. The homeowner's attorney also moved for a directed verdict. How should the court rule on the directed verdict motions? (A) Deny the electrical company's motion and grant the homeowner's motion for a directed verdict, because a short in the wiring caused the homeowner's injuries. (B) Deny the electrical company's motion and grant the homeowner's motion for a directed verdict, because the company failed to rebut the presumption of negligence that the homeowner has established. (C) Deny the homeowner's motion and grant the electrical company's motion for a directed verdict, because the wire could have been damaged by another contractor. (D) Deny both directed verdict motions, because the homeowner has presented enough evidence to submit the case to the jury.

C The court should grant the electrical company's motion for a directed verdict because the homeowner has not established a prima facie case of negligence on the company's part. The homeowner has established that the electrical company owed a duty to him and that he has suffered harm from the fire caused by the short in the wiring. However, he has not established that the company breached any duty to him. While breach of duty is ordinarily a question for the trier of fact, a plaintiff's failure to offer any evidence on that element of the prima facie case will permit a directed verdict for the defendant. Under certain circumstances, the fact that a particular injury occurred may itself establish or tend to establish a breach of duty owed, permitting the trier of fact to infer the defendant's liability. This is the doctrine of res ipsa loquitur ("the thing speaks for itself"). However, for the doctrine to apply, the plaintiff must show that (i) the accident causing his injury is the type that would not normally occur unless someone was negligent; (ii) the negligence was attributable to the defendant; and (iii) the injury was not attributable to the plaintiff. The second requirement can often be satisfied by showing that the instrumentality causing the injury was in the exclusive control of the defendant. Here, however, the wiring was exposed to work done by other contractors in installing a chimney and a hot water pipe nearby and putting up the walls, and the homeowner has offered no evidence that the cut in the outer sheath of the wiring was present when the electrical company finished its work. Instead, the fact that the wiring had been approved by the building inspector suggests that the wiring was intact when the electrical company finished. Given these facts, the homeowner has not presented evidence that the negligence was attributable to the defendant. Since res ipsa loquitur does not apply and no other evidence of breach of duty was established, the electrical company's motion for a directed verdict should be granted. (C) is therefore correct, and (B) and (D) are incorrect. (B) is also incorrect because the homeowner's motion for a directed verdict would be denied even if he had established res ipsa loquitur. Establishing res ipsa loquitur merely creates a permissible inference of negligence; it does not create a presumption of negligence. Where the res ipsa loquitur element has been proved, the plaintiff has established a prima facie breach of duty on the defendant's part and no directed verdict may be given for the defendant. However, it does not require the defendant to present evidence to rebut a presumption. The trier of fact is free to accept the inference of negligence that has been created and find for the plaintiff or reject the inference of negligence and find for the defendant, even if the defendant offers no other evidence on the issue. Thus, the court would not grant the homeowner's motion for a directed verdict even if he had established res ipsa loquitur. (A) is incorrect because the electrical company is not strictly liable for the short in the wiring. The homeowner's failure to offer some evidence of negligence on the part of the electrical company will allow the electrical company to prevail.

After enjoying a wonderful meal in a restaurant, a diner went into the kitchen through a door marked "employees only" to personally compliment the chef. However, before he could get the attention of the chef, he slipped on a puddle of bright yellow grease that had congealed on the floor by the stove. He fell, hitting his head and sustaining a severe head injury. If the diner sues the restaurant for damages in a jurisdiction following the traditional rules for landowners and possessors of land, is he likely to recover? (A) Yes, because the restaurant is a place of public accommodation and breached its duty of care owed to its patrons. (B) Yes, because a restaurant employee could have discovered the dangerous condition of the floor by making reasonable inspections. (C) No, because patrons were not allowed in the kitchen. (D) No, because the puddle of grease was visible on the floor.

C The diner is not likely to recover because the restaurant did not permit patrons in the kitchen. In jurisdictions following the traditional rules for landowners and possessors of land, the duty owed by an owner or occupier of land to those entering the land depends on whether the entrant is characterized as a trespasser, licensee, or invitee. Here, the diner was an invitee of the restaurant while he was dining at the restaurant. However, a person loses his status as an invitee if he exceeds the scope of the invitation-if he goes into a portion of the premises where his invitation cannot reasonably be said to extend. Here, the diner lost the status of an invitee when he entered the kitchen; he became, at best, a licensee, perhaps even a trespasser, because patrons were not permitted in the kitchen. While a landowner owes no duty to an undiscovered trespasser, he owes a discovered trespasser the duty to warn of or make safe artificial conditions known to the landowner that involve a risk of death or serious bodily harm and that the trespasser is unlikely to discover. For a licensee, the duty extends to all dangerous conditions that create an unreasonable risk of any harm to the licensee. Had the diner been an invitee, he could have argued that under the duty owed to invitees to make reasonable inspections, a puddle of grease that had time to congeal should have been discovered and cleaned up, or at least been the subject of a warning. Here, however, there are no facts to suggest that anyone in the kitchen knew of the diner's presence or the puddle of grease, even though it had congealed; hence, the facts do not establish a breach of the duty to warn discovered trespassers or licensees of dangerous conditions. (A) is incorrect. While places of public accommodation have an affirmative duty to use reasonable care to aid or assist their patrons, that duty rule does not alter the duty rules pertaining to the condition of the land, which are based on the status of the person on the premises (as discussed above). (B) is incorrect because the failure to exercise reasonable care to discover a dangerous condition breaches a duty owed only to invitees, as discussed above. However, the diner was no longer an invitee when he entered the kitchen area. He was at most a licensee, for whom the duty to make reasonable inspections does not apply. (D) is incorrect. While a duty to warn does not exist where the dangerous condition is so obvious that the invitee should reasonably have been aware of it, just the fact that the grease was visible on the floor does not establish this. "Obviousness" is determined by all of the surrounding circumstances; e.g., one whose attention may have been directed elsewhere may recover even though the condition was visible. In any case, even if the puddle were not visible, the diner would not be able to recover because he did not have invitee status when he entered the kitchen.

Which of the following may prevent establishing causation against a manufacturer in a strict products liability action? A The retailer's labeling of the product as its own B The destruction of the product because of its dangerous defect C The failure of a retailer to take action after discovering a dangerous defect D The negligent failure of a retailer to discover a dangerous defect

C The failure of a retailer to take action after discovering a dangerous defect may prevent establishing causation against a manufacturer in a strict products liability action. The same concepts of proximate cause that govern negligence and strict liability actions are applicable to strict liability actions for defective products. As with products liability cases based on negligence, the negligent failure of a retailer to discover a dangerous defect does not cut off the supplier's strict liability. On the other hand, when the intermediary's conduct becomes something more than ordinary foreseeable negligence, it becomes a superseding cause. Hence, the conduct of a retailer who discovered a dangerous defect and then took no action (such as alerting the manufacturer, warning the consumer, or removing the product from sale) constitutes more than ordinary foreseeable negligence and may cut off the manufacturer's liability. The destruction of the product because of its dangerous defect does not prevent establishing actual cause. If the product has been destroyed, the plaintiff may rely on an inference that this type of product failure ordinarily would occur only as a result of a product defect. The retailer's labeling of the product as its own will not affect the liability of the manufacturer.

Which of the following persons is considered a licensee of the landowner? (A) A member of the public visiting a free museum. (B) A health inspector determining whether to renew the landowner's food service license. (C) A friend of the landowner invited in after giving the landowner a ride home. (D) A trespasser who the landowner knows regularly crosses over a section of the landowner's land.

C The friend of the landowner is a social guest and therefore a licensee. Under traditional landowner liability rules, a licensee is one who enters on the land with the landowner's permission, express or implied, for her own purpose or business rather than for the landowner's benefit. Even though a host may also derive some benefit from the presence of a social guest, including receiving the performance of minor services by the guest, this does not make the guest an invitee. A trespasser who the landowner knows regularly crosses over a section of the landowner's land is an anticipated trespasser and not a licensee. The duty owed to discovered and anticipated trespassers is not as high as the duty owed to licensees. A health inspector determining whether to renew the landowner's license is not a licensee. Persons who enter the premises for a purpose connected with the landowner's business are invitees and are owed a higher duty. Similarly, a member of the public visiting a free museum is an invitee. Invitees include those who enter the premises for a purpose for which the land is held open to the public.

A college student owned a very popular video game system that was out of stock in most stores. He agreed to let his friend use the system for a few days, on condition that he return the system by the weekend because the student was hosting a small party. On the morning of the party, the friend still had not returned the game system, so the student went to the friend's apartment and demanded it back. The friend refused, so the student grabbed the system and wrestled it out of the friend's hands. If the friend sues the student for battery, will he recover on this claim? (A) No, because the student used reasonable force in attempting to seize the game system. (B) No, unless the student proves that the friend's delay in the return of the game system was unreasonable. (C) Yes, because the student had originally agreed to lend the game system to the friend. (D) Yes, because the student had to give the friend a reasonable period of time after demand in which to return the game system.

C The friend will recover for battery because the student did not have the right to use force. The defense of recapture of chattels is limited by the circumstances of the original dispossession. When another's possession of the owner's chattel began lawfully, the owner may use only peaceful means to recover the chattel. Force may be used to recapture a chattel only when in "hot pursuit" of one who has obtained possession wrongfully (e.g., by theft). Here, the friend's initial possession of the game system was a bailment, because the student consented to his borrowing it. Thus, the student is not entitled to use force to recover it, and his wrestling it away from the friend constituted the requisite harmful or offensive contact to make the student liable for battery. (A) is incorrect. One who is entitled to use force to recapture chattels is only permitted to use reasonable force, but here the student is not entitled to use any force at all because the friend's initial possession of the game system was lawful. (B) is incorrect because it is not relevant whether the friend's delay in returning the game system was unreasonable; the student is not entitled to use force because he lent the system to the friend originally. (D) is incorrect because the requirement that a timely demand to return the chattel must precede the use of force applies only if the owner of the chattel is entitled to use force. Here, as discussed above, the student did not have the right to use force.

A golfer and her instructor were playing golf in a foursome when the golfer became very annoyed with critical comments made by the instructor. To show the other golfers in the group how annoyed she was with her instructor, the golfer stood a few yards behind him while the instructor was teeing off and swung a club at him. The instructor, who was focusing on his shot, was not within range of the club but unfortunately the club slipped out of the golfer's hands and struck the instructor in the head, injuring him. If the instructor brings a battery action against the golfer, will he recover? (A) Yes, because the golfer acted intentionally and caused harmful contact to her instructor. (B) Yes, because the golfer intended to cause the instructor reasonable apprehension of imminent harmful contact. (C) No, because the golfer did not intend to cause harmful or offensive contact. (D) No, unless the golfer acted unreasonably in swinging the club at her instructor.

C The golfer will not be liable because she did not intend to cause harmful or offensive contact. The prima facie case for battery has the following elements: (i) an act by the defendant that brings about harmful or offensive contact to the plaintiff's person; (ii) intent on the part of the defendant to bring about harmful or offensive contact to the plaintiff's person; and (iii) causation. Here, the golfer did not have the intent to cause harmful or offensive contact. Hence, she will not be guilty of battery. (A) is incorrect because even though the golfer had the intent to swing the club, she did not have the intent required for battery-to cause harmful or offensive contact to another. (B) is incorrect because the facts do not support an intent to cause an assault. Under the transferred intent doctrine, an intent to cause an assault (intent to cause apprehension of imminent harmful or offensive contact) will satisfy the intent requirement for battery when the other elements of battery are present. Here, however, the golfer was standing behind the instructor and was intending only to show the other golfers how annoyed she was. No intent to commit assault is apparent here. (D) is incorrect because it describes a negligence standard. The instructor may be able to recover against the golfer in a negligence cause of action if the golfer acted unreasonably in swinging the club, but this does not establish intent for a battery action.

A barge transporting toxic chemical waste down a river lost its navigation control when its propeller broke without warning. The barge struck a dinner cruise ship, injuring several passengers, but fortunately the toxic waste did not leak from the barge. In an action alleging strict liability against the barge operator, an injured cruise ship passenger established the above facts and presented evidence of her injuries. The barge operator presented evidence that the propeller failure was caused by a hidden defect in the metal of one of the propeller blades that could not be detected by routine inspection. The propeller was manufactured by a subcontractor of the shipbuilder that assembled the barge, and these types of propellers had not previously caused any problems. The barge operator also presented evidence that the local authorities were supposed to restrict other ships' access to the river while the barge was in transit but had failed to do so. The jurisdiction has decided that the transport of toxic chemical waste is an abnormally dangerous activity. In this action, is the passenger likely to prevail? (A) Yes, because the propeller was in a defective condition that made it unreasonably dangerous. (B) Yes, because the barge operator was engaged in an abnormally dangerous activity. (C) No, because the injury did not arise from the dangerous aspect of the activity. (D) No, because the negligence of the local authorities in failing to restrict access to the river while the barge was in transit was a superseding cause of the passenger's injuries.

C The passenger is not likely to prevail in a strict liability action because her injury did not arise from the abnormally dangerous aspect of the barge operator's activity. The barge operator's transport of toxic waste has been determined to be an abnormally dangerous activity. An activity is considered abnormally dangerous when: (i) it creates a foreseeable risk of serious harm even when reasonable care is exercised by all actors; and (ii) the activity is not a matter of common usage in the community. However, the scope of liability extends only to the dangers that would be anticipated from the activity involved; strict liability does not apply to harms that were not caused by the normally dangerous aspect or propensity of the activity. Here, the barge operator's activity is subject to strict liability because of the danger of toxic waste escaping from the barge, but not from a crash by itself. Because the passenger's injuries were not caused by exposure to toxic waste, strict liability does not apply. The barge operator would be liable for the injuries from the crash only if the passenger established negligence. (A) is incorrect because it states the standard for strict liability for defective products, which requires that the defendant be a commercial supplier of the defective product, i.e., someone in the chain of distribution of the product. Here, the barge operator is not a commercial supplier of the propeller that failed; rather, it is the purchaser or consumer of the propeller. (B) is incorrect because, as discussed above, strict liability does not apply to the harm that occurred here. (D) is incorrect. It is questionable whether the negligence by the local authorities could be considered an intervening force, which must come into play after the culpable conduct by the defendant. Even if it were an intervening force, it likely would not be considered so extraordinary as to be an unforeseeable intervening force. Hence, it would not constitute a superseding force that would break the causal connection between the passenger's injury and the barge operator's actions.

While driving his car down the road, the defendant, who had no history of heart problems, experienced a heart attack. The defendant's car crossed the center line of the highway, in violation of a motor vehicle statute, and headed directly at a car driven by the plaintiff that was exceeding the speed limit. The plaintiff, seeing the defendant's car heading toward him, swerved to avoid the collision. In so doing, the plaintiff's car spun out of control and crashed into a ditch, causing the plaintiff injury. The plaintiff brought suit against the defendant for the injuries sustained in the accident. The jurisdiction retains traditional contributory negligence rules. Will the plaintiff prevail? (A) Yes, because the defendant's act was a substantial factor in causing the plaintiff's car to swerve. (B) Yes, because the defendant violated a statute by crossing the center line. (C) No, because the defendant had no prior history of heart trouble. (D) No, because the plaintiff was exceeding the speed limit.

C The plaintiff will not prevail because the defendant had no history of heart problems. For the plaintiff to prevail in a claim against the defendant, the plaintiff must show (i) duty, (ii) breach of duty, (iii) actual and proximate cause, and (iv) damages. While drivers owe a duty of care to other drivers on the road, a driver would not be deemed to have breached that duty if he had a surprise heart attack while driving, given that he had no history of heart trouble. (A) is wrong because the fact that the defendant's car was a cause of the accident does not establish breach of duty. (B) is wrong because violation of an applicable statute, which may sometimes establish breach of duty, will be excused where compliance was impossible. Here, because the defendant suffered a sudden heart attack, he was unable to comply with the motor vehicle statute. (D) is wrong. The facts do not establish whether the accident would have happened regardless of the plaintiff's contributory negligence, but the trier of fact will not reach that issue because the plaintiff will not be able to establish the prima facie case.

Which of the following is NOT a type of intervening force that is almost always foreseeable in a proximate cause analysis?

Criminal acts of third persons

A state child safety statute required children under eight years of age to be in a government-approved car seat when riding in a motor vehicle. A father was driving to a ballgame with his seven-year-old child, who was buckled in the back seat with a regular seat belt. The father did not notice when the child unbuckled himself and started climbing into the front seat. The child grabbed the steering wheel "to help daddy steer," causing the car to swerve into the other lane and collide with another motorist's car. The motorist was seriously injured from the collision; the father and his child were unhurt. The motorist sued the father to recover damages for her injuries. At trial, the motorist presented evidence of the statute, her injuries, and the facts stated above. At the conclusion of the proofs, both parties moved for a directed verdict. How should the trial judge proceed? (A) Grant the motorist's motion, because the father's violation of the statute constituted negligence per se. (B) Grant the father's motion, because the motorist offered no evidence that the statute was intended to prevent the harm that occurred. (C) Deny both motions and submit the case to the jury, because the jury could find that the father is liable for his child's negligent conduct. (D) Deny both motions and submit the case to the jury, because the jury could find that the father breached his duty of care owed to the motorist.

D The court should deny both motions and submit the case to the jury, because the jury could find that the father was negligent in not preventing his child from grabbing the steering wheel. Under ordinary negligence principles, the father owed a duty to other motorists to maintain control of his vehicle. The jury could find that the father breached this duty of care by not noticing when his child unbuckled the seat belt and started climbing into the front seat, and not preventing the child from grabbing the steering wheel. Hence, the case should go to the jury for a determination of whether the father was negligent. (A) is incorrect because the facts do not establish that the statutory standard of care is applicable to these facts. The precise standard of care in a common law negligence case may be established by proving that a statute providing for a criminal penalty applies to the particular case. If that is done, the statute's more specific duty will replace the more general common law duty of care. Violation of the statute establishes negligence per se-a conclusive presumption of duty and breach of duty; the plaintiff must then establish causation and damages to complete the prima facie case of negligence. To prove that the statutory standard applies, the plaintiff must show that (i) she is in the class intended to be protected by the statute, and (ii) the statute was designed to prevent the type of harm that occurred. Here, the statute likely was intended primarily to protect children from injuries caused by not being properly restrained in a vehicle involved in a collision. The motorist has presented no evidence that the statute was intended to protect her from the harm that she suffered. (B) is incorrect because even though the motorist did not establish that the statute applies to her claim, she has presented facts sufficient to allow the jury to find that the father breached the common law duty of care that he owed to other motorists. (C) is incorrect because the father is not vicariously liable for his child's negligent conduct at common law. Any liability of the father in this case would arise from his own potential negligence in failing to control his child while driving.

A boy was playing softball in a neighborhood park when a ball was hit over the fence and into a neighbor's yard. The boy knocked on the neighbor's door and obtained permission from her to retrieve the ball from her yard. As he bent to retrieve the ball in some bushes, the boy brushed against an exposed electric wire that was partially hidden by the bushes and received a severe electric shock and burns. The neighbor had failed to maintain the bushes, allowing them to become overgrown, and was not aware of the exposed wire. If the boy sues the neighbor in a jurisdiction that applies the traditional rules for landowners and possessors of land, what is the likely result? (A) The neighbor is liable because the boy entered with her permission. (B) The neighbor is liable because she failed to repair a dangerous condition on her property. (C) The neighbor is liable because she failed to reasonably inspect the property, which would have made her aware of the dangerous condition of the wire. (D) The neighbor is not liable because she did not know of the condition of the wire.

D The neighbor is not liable because she did not know of the condition of the wire and the boy was a licensee. In jurisdictions that distinguish between invitees and licensees, a licensee is a person who enters land with the owner's permission, for his own purpose or business rather than for the owner's benefit. The owner or occupier of land owes the licensee a duty to warn of or make safe a dangerous condition known to the owner or occupier that creates an unreasonable risk of harm to the licensee and that the licensee is unlikely to discover. However, the owner or occupier has no duty to a licensee to inspect for defects nor to repair known defects. The boy was a licensee because he entered the neighbor's land with her permission for his own purpose (retrieving the ball) rather than for any benefit of the neighbor's. The exposed electric wire created an unreasonable risk of death or serious injury to the boy as he reached into the bushes. Because the neighbor did not know of the presence and condition of the wire, she had no knowledge of any risk of harm to the boy. Thus, no duty to warn the boy of the wire was triggered. (A) is incorrect because the mere fact that the neighbor gave the boy permission to enter her land will not subject her to liability for his injuries incurred thereon. The neighbor is not strictly liable for injuries to a licensee, but only for any injuries caused by a breach of her duty to warn of dangerous conditions known to her and that the licensee is unlikely to discover. (A) would impose liability even where the neighbor had no knowledge of the condition of the wire. (B) is incorrect because, as noted above, an owner of land does not owe a duty to a licensee to repair defects or dangerous conditions. Likewise, (C) is incorrect because the owner of land is not under a duty to a licensee to make an inspection to discover defects or dangerous conditions.

Under the doctrine of respondeat superior, an employer: A May be liable for negligently supervising an employee B Is liable for the tortious frolic of an employee C Is liable for all tortious acts committed by his employees D May be liable for an intentional tortious act committed by an employee

D Under the doctrine of respondeat superior, an employer is vicariously liable for tortious acts committed by his employees if the tortious acts occur within the scope of the employment relationship. Under this doctrine, an employer may be liable for an intentional tortious act committed by an employee. While the general rule is that intentional tortious conduct by employees is not within the scope of employment, courts will find intentional tortious conduct to be within the ambit of this relationship when (i) force is authorized in the employment; (ii) friction is generated by the employment; or (iii) the employee is furthering the business of the employer. An employer is not liable for ALL tortious acts committed by his employees ; the acts must be within the scope of the employment relationship. An employer is NOT liable for the tortious frolics of his employees. An employee on a delivery or business trip for his employer may commit a tort while deviating from the employer's business to run a personal errand. If the deviation was minor in time and geographic area, the employee will still be considered to be acting within the scope of employment, but a more major deviation is a "frolic" for which the employer would not be liable. While it is true that an employer may be directly liable for negligent supervision, the employer will NOT be liable for negligently supervising an employee under the respondeat superior doctrine, which only imposes vicarious liability. An employer's liability for negligent supervision will be direct liability rather than vicarious liability.

On the way home from a nightclub, a passenger began yelling at the designated driver claiming that he was not taking the best route back to her house. The driver disagreed and contended that his route was the quickest. The passenger impulsively grabbed the steering wheel, causing the car to swerve and strike a pedestrian, injuring him. At trial, the pedestrian established that the driver's license had expired the day before the accident. The driver's traffic record qualified him for an automatic renewal of his driver's license, but he had forgotten to submit it in time. A statute in the jurisdiction makes it an offense to drive a vehicle on any public road in the state without a valid driver's license. Will the pedestrian prevail? (A) Yes, because the driver violated a statute by driving without a valid license. (B) Yes, because the driver failed to control his passenger. (C) No, because the driver did not start the argument. (D) No, because the passenger's action was the proximate cause of the injury.

D he pedestrian will not prevail because the passenger's grabbing of the wheel is the negligent conduct that caused the pedestrian's injuries. To establish a prima facie case for negligence, the following elements must be proved: (i) the existence of a duty on the part of the defendant to conform to a specific standard of conduct for the protection of the plaintiff against an unreasonable risk of injury; (ii) breach of that duty by the defendant; (iii) the breach of the duty by defendant was the actual and proximate cause of plaintiff's injury; and (iv) damage to plaintiff's person or property. Here, the driver's actions were an actual cause of the pedestrian's injury because, but for the driver's driving and the passenger's grabbing the steering wheel, the injury to the pedestrian would not have happened. However, the driver's actions were not a proximate cause of the injury because the passenger's grabbing of the steering wheel was a superseding intervening force. A superseding force is one that serves to break the causal connection between the initial wrongful act and the ultimate injury, and itself becomes a direct immediate cause of such injury. Thus, the first actor would be relieved of liability from the consequences of his antecedent conduct. The passenger's conduct in suddenly grabbing the steering wheel was an unforeseeable intervening force creating an unforeseeable harmful result, and thus constituted a superseding force. Consequently, the driver would be relieved of any negligence liability since the passenger's actions were the proximate cause of the accident. (A) is incorrect. A clearly stated specific duty imposed by a statute may replace the more general common law duty of due care when (i) the plaintiff is within the class to be protected by the statute, and (ii) the statute was designed to prevent the type of harm suffered. The statute probably does not apply here because it is intended to keep unsafe drivers off the streets, and there is no indication that the driver is an unsafe driver, or that any driver could have prevented the injury when the passenger grabbed the steering wheel. Even if the statutory standard were applicable, a violation means only that plaintiff will have established a conclusive presumption of duty and breach of duty. It does not, however, establish causation or damages. Here, the fact that the driver does not have a valid license is not the proximate cause of the pedestrian's injury, as discussed above. (B) is incorrect because the driver had no way of knowing that the passenger would grab the steering wheel. The driver's conduct will be measured against that of the ordinary, prudent, reasonable person who drives a vehicle. An ordinary, prudent, reasonable person would not have foreseen that one of his passengers would impulsively grab the steering wheel, and therefore there are no special safety precautions that the driver should have taken as part of his duty of care toward pedestrians. (C) is incorrect because, even if the driver had started the argument, this would not justify the passenger's grabbing of the steering wheel. In either case, the passenger's actions rather than the driver's would be considered the proximate cause of the pedestrian's injuries.

In contrast to products liability cases based on negligence, those based on strict liability do NOT: A Impose liability when an intermediary negligently failed to discover the defect. B Prohibit recovery of solely economic losses. C Require that suppliers have an opportunity to inspect. D Require an injured bystander to be foreseeable.

Do not require that suppliers have an opportunity to inspect. Thus, for a case based on the sale of a defective product, a retailer in a strict liability action may be liable for a manufacturing or design defect simply for being a commercial supplier of that defective product, even if it had no opportunity to inspect the manufacturer's product before selling it. In a negligence action, the supplier's negligence must be proved. Products liability cases based on negligence and those based on strict liability both require that an injured bystander be foreseeable. While privity is not required in these cases, and bystanders are protected and may bring a claim under either theory, they must be foreseeable. Liability under these theories applies to foreseeable plaintiffs. Products liability cases based on negligence and those based on strict liability both prohibit recovery of solely economic losses. The types of damages recoverable under both theories are the same: personal injury and property damages. Economic loss cannot be the sole damage claim. As under claims based on negligence, those based on strict liability will impose liability even though an intermediary negligently failed to discover the defect. The same concepts of proximate cause govern negligence and strict liability actions. The negligent failure of an intermediary to discover a defect is not a superseding cause and does not cut off the supplier's strict liability. However, if the intermediary's conduct becomes something more than ordinary foreseeable negligence, then it does become a superseding cause.

T or F: Strict liability for wild animals does not include liability for the harm that results when a person is attempting to flee from what is perceived to be a dangerous animal.

False. It does

Negligence - Duty

Foreseeability 1. Owe a duty to foreseeable victims, you don't owe a duty to unforeseeable victims. Therefore unforeseeable victims always lose negligence claim on bar exam. 2. Palsgraff- outside the zone of danger, she lost the case because she was geographically very far away from the incident a. Foreseeable victims are people that are near you. b. Unforeseeable victims are people that are far away from you. ii. Reasonably Prudent Person - a general duty of care is imposed on all human activity. When a person engages in an activity, he is under a legal duty to act as an ordinary, prudent, reasonable person. 1. Reasonably prudent person = has no physical attributes, no race, no gender, no height, no age, etc. 2. It is an OBJECTIVE standard a. Rigid, inflexible, and objective standard b. Ex: mentally disabled person still has to follow the RPP standard, even if it is impossible for him to do so. Mentally ill, novices, inexperienced people, etc. 3. Exceptions: (a) If a person has a superior skill or superior knowledge, then we will hold the RPP to that superior skill or knowledge / added knowledge i. Ex: race car driver who drives to the grocery store on a normal day, if he gets into a skid, he would be expected to know how to handle the car in such a skid because of his superior skill or knowledge ii. Ex 2: if a stupid person knew ahead of time that there is a shrub that blocks the visibility of this part of the road, then he has added knowledge even though he is dumb, then can be held liable because RPP takes that added knowledge into an account (b) where relevant, defendant's physical characteristics are taken into account i. Ex: where a defendant is blind, then RPP takes that into account - we expect a RPP blind person not to see and also to do the things blind people do, don't drive, use a cane, etc

Self Defense - property analysis in intentional torts

General self-defense principle: When a person has reasonable grounds to believe that he is being, or is about to be, attacked, he may use such force as is reasonably necessary for protection against the potential injury. Generally, one may use reasonable force to prevent the commission of a tort against her property. 1. Request to Desist Usually Required - a request to desist must precede the use of force, unless the circumstances make it clear that the request would be futile or dangerous. 2. Effect of Mistake - reasonable mistake is allowed as to the property owner's right to use force in defense of property where the mistake involves whether an intrusion has occurred or whether a request to desist is required. However, mistake is not allowed where the entrant has a privilege to enter the property that supersedes the defense of property right (see 4 below). In such a case the property owner is liable for mistakenly using force against a privileged entrant unless the entrant himself intentionally or negligently caused the mistake (e.g., by refusing to tell the property owner the reason for the intrusion). 3. Limited to Preventing Commission of Tort - defense of property is limited to preventing the commission of tort against the defendant's property. Thus, once the defendant has to been permanently dispossessed of the property and the commission of the tort is complete, she may not use force to recapture it. However, where one is in "hot pursuit" of someone who wrong-fully dispossessed her of her property, the defense still operates because the other is viewed as still in the process of committing the tort against the property. 4. Superseded by other Privileges - whenever an actor has a privilege to enter upon the land of another because of necessity, right of reentry, right to enter upon another's land to recapture chattels, etc., that privilege supersedes the privilege of the land possessor to defend her property How much Force may be used? One may use reasonable force to defend property. However, she may not use force that will cause death or serious bodily harm. (of course, if the invasion of property also entails a serious threat of bodily harm to the owner, she may then invoke the defense of self-defense and use deadly force). Further, one may not use indirect deadly force such as a trap, spring gun, or vicious dog when such force could not lawfully be directly used, e.g., against a mere trespasser.

A doorman negligently locked a door that an office worker was intending to use to exit an office building, so the worker was forced to use a different exit. As she stepped onto the sidewalk outside the building, a car careened out of control on the street and jumped the curb. The car struck and injured the worker and then drove off. The driver was not found. The worker brought suit against the doorman, seeking damages for her injuries. At trial, the parties stipulated that the doorman was negligent in locking the door and that the worker suffered injuries when she was struck by the car. The worker also established that if she had exited from the door she was intending to, she would not have been struck by the car. At the end of the worker's case, the doorman moved for a directed verdict in his favor. How should the judge rule?

Grant the motion, because the car was an unforeseeable intervening force. The court should grant the motion because the evidence establishes that the car was a superseding force that cut off the doorman's liability for his negligence under proximate cause principles. The general rule of proximate cause is that the defendant is liable for all harmful results that are the normal incidents of, and within the increased risk caused by, his acts. An indirect cause case is one where the facts indicate that a force came into motion after the time of defendant's negligent act and combined with the negligent act to cause injury to the plaintiff. Whether an intervening force will cut off the defendant's liability for the plaintiff's injury and be deemed superseding is determined by foreseeability. Here, nothing in the facts suggests that a car jumping the curb was a foreseeable consequence of the doorman's negligently locking the door. Hence, the judge should grant the motion because the worker has failed to establish the proximate cause element of his prima facie case. (A) is wrong because the doorman was also an actual cause of the worker's injuries-but for the doorman's negligence, the worker would not have been on the sidewalk where the car jumped the curb. (C) is wrong because it establishes only actual cause. A directed verdict is appropriate because no evidence establishes the proximate cause element of the worker's case. (D) is wrong because the facts do not establish foreseeability. While the doorman's negligence was a concurring actual cause of the worker's injury, it was not a proximate cause because the injury that occurred was unforeseeable.

Which of the following is NOT a prerequisite for the plaintiff to rely on the doctrine of res ipsa loquitur (A) The injury was not attributable to the plaintiff or any third person. (B) The defendant had actual possession of the instrumentality causing the injury. (C) There is evidence connecting the defendant with the negligence. (D) The accident causing the injury is the type that would not normally occur unless someone was negligent.

It is not necessary for application of res ipsa loquitur to show that the defendant had actual possession of the instrumentality causing the injury, even though this may be one way to connect the defendant with the negligence that occurred. The circumstantial evidence doctrine of res ipsa loquitur deals with those situations where the fact that a particular injury occurred may itself establish or tend to establish a breach of duty owed. Res ipsa loquitur requires that: 1. The accident is of a type that normally does not occur in the absence of someone's negligence; 2. The evidence connects the defendant to the negligence (i.e., this type of accident ordinarily happens because of the negligence of someone in the defendant's position); and 3. The injury was not attributable to the plaintiff or any third person.

If a statute providing for a criminal penalty is applicable to a common law negligence case, a clearly stated specific duty imposed by the statute will replace the more general common law duty of care. Most courts hold that violation of an applicable statute __________.

Most courts still adhere to the rule that violation of an applicable statute is "negligence per se." This means that the plaintiff will have established a conclusive presumption of duty and breach of duty by showing a violation of the statute. (The plaintiff still must establish causation and damages to complete the prima facie case for negligence.) For a statute to apply in a negligence case: (i) The plaintiff must show that she is in the class intended to be protected by the statute; and (ii) The plaintiff must show that the statute was designed to prevent the type of harm that the plaintiff suffered

Battery

Plaintiff must show that 1) D committed a harmful or offensive contact 2) That contact has to be with the P's person - Offensive = unpermitted - plaintiff's person includes anything that person is touching

A consumer purchased a grass trimmer from a hardware store. He took it out of the box and assembled it according to the instructions. He noticed that there were bolts and screws left over and some joints that could have accepted additional fasteners, but he just discarded the extra hardware. As he was using the trimmer, the housing came apart and a hard piece of plastic flew off. His neighbor, who was standing nearby, was struck in the eye by the piece of plastic and suffered permanent injuries. The neighbor sued the hardware store and the manufacturer of the trimmer in a strict liability action. Through discovery, it was determined that the instructions omitted a critical step in the assembly process that would have used the extra hardware, which is why the housing came apart, and that the manufacturer had received some complaints about the instructions previously. The hardware store had no knowledge of any complaints regarding any of the manufacturer's products. As to the hardware store, the neighbor will: Recover or not recover? Why?

Recover (B) The neighbor will recover against the hardware store. In a products liability action based on strict liability, the plaintiff need show only (i) the defendant is a commercial supplier, (ii) the defendant produced or sold a defective product, (iii) the defective product was an actual and proximate cause of the plaintiff's injury, and (iv) the plaintiff suffered damages to person or property. Here, the hardware store is in the chain of supply of the product. The product was defective because the instructions omitted a critical part of the assembly process. Finally, the omission was an actual and proximate cause of the neighbor's injury, allowing him to recover against the hardware store. As indicated by choice (B), an intermediary's negligent failure to recognize the danger does not cut off the supplier's strict liability. Answer (A) is incorrect because the manufacturer's awareness of the faulty instructions does not affect whether the hardware store will be liable. The hardware store was not aware of any problems with the product, but it is nevertheless liable as a commercial supplier. Answer (C) is incorrect. The neighbor, as a bystander, is within the foreseeable zone of danger and is therefore a foreseeable plaintiff who can recover in this action. Answer (D) is incorrect because the fact that the hardware store was not at fault and had no opportunity to inspect is irrelevant. It is liable because it is a commercial supplier of a defective product and the neighbor is suing under a strict liability theory

The driver of a tanker truck was transporting radioactive waste from a nuclear power plant to a permanent storage facility in a remote western region of the United States. After driving all night, the driver fell asleep at the wheel and the truck crossed over the center line, off the road, and onto a homeowner's property, coming to rest after crashing into several glass cases containing the homeowner's collection of poisonous snakes, the keeping of which was permitted by local ordinance. When the driver exited the truck, he was bitten on the leg by one of the poisonous snakes and became seriously ill. The driver brought an action against the homeowner for his injuries. The parties stipulated to the above facts, and that the driver violated a state statute by driving off of the road. Both parties moved for judgment as a matter of law on the liability issue. How should the court rule? (A) Grant the driver's motion and deny the homeowner's motion, because the homeowner is strictly liable for the injury caused by the snake. (B) Deny the driver's motion and grant the homeowner's motion, because the driver was a trespasser on the homeowner's property. (C) Deny the driver's motion and grant the homeowner's motion, because the driver's violation of the state statute establishes contributory negligence as a matter of law. (D) Deny both parties' motions, because both parties were engaged in an activity for which strict liability is imposed.

The court should grant the homeowner's motion for judgment as a matter of law because the driver has not established a prima facie case against the homeowner. An owner of wild (dangerous) animals is strictly liable for injuries caused by those animals as long as the person injured did nothing, voluntarily or consciously, to bring about the injury. However, strict liability generally is not imposed in favor of undiscovered trespassers against landowners in the absence of negligence, such as when the landowner knows that the trespassers are on the land and fails to warn them of the animal. Here, despite the fact that the driver did not intend to enter the homeowner's land (and thus would not be liable for the intentional tort of trespass), his status on the homeowner's land is that of a trespasser rather than a licensee or invitee. The driver has presented no evidence of negligence on the homeowner's part and therefore has not established a prima facie case against the homeowner. (A) is wrong because, as discussed above, the homeowner is not strictly liable to the driver because the driver was a trespasser. (C) is incorrect because the driver will not prevail regardless of whether he was contributorily negligent, because he cannot establish a prima facie case against the homeowner in either negligence or strict liability. (D) is incorrect for several reasons: While the driver's transport of radioactive waste may have been an abnormally dangerous activity, that danger had nothing to do with the accident that occurred. Furthermore, the fact that the driver may have been engaged in an abnormally dangerous activity would not prevent him from recovering damages from another tortfeasor if he established the requisite prima facie case. Finally, the fact that the parties were engaged in activities potentially creating strict liability has nothing to do with whether issues of fact regarding liability still exist that would require denying both motions and going to trial.

Which of the following circumstances has no bearing on whether an employer will be vicariously liable for an employee's intentional tort? What does have bearing?

The employee was negligently supervised by the employer. Vicarious liability is derivatively imposed liability. The fact that the employee was negligently supervised by the employer has no bearing on whether the employer will be vicariously liable for the employee's intentional torts; rather, it will make it more likely that the employer will be directly liable for its own negligence. In contrast, the fact that force was authorized in the employment, friction was generated by the employment, or the employee was furthering the business of the employer make it more likely that the employee's intentional torts will be found to be within the scope of employment.

A developer constructed several small stores in a commercial district. She received a bid from a contractor to install awnings on the front windows of the stores. The developer had heard that the contractor did shoddy work, but the price was right and the contractor expressly assumed all of the risk of any liability. The developer subsequently sold one of the stores to a barber. A few months later, an awning collapsed without warning, injuring a customer who was about to enter the barbershop. An investigation by the building inspector revealed that the awning collapsed because the brackets used by the contractor were cheaper and weaker than the required brackets, although they looked the same. The developer and the contractor are now both bankrupt. If the customer sues the barber for his injuries, is the customer likely to prevail?

The customer is not likely to prevail because the barber had no opportunity to oversee the contractor's work. A property owner owes a duty to those adjacent to the premises to take due precautions to protect them from dangerous conditions, and a business owes its customers a duty to make reasonable inspections to discover and make safe any dangerous conditions. Further, that duty cannot be delegated to an independent contractor; the owner remains vicariously liable. However, nothing in the facts establishes that the barber breached his duty to the customer. There was nothing wrong with the brackets evident from a reasonable inspection, and the awning collapsed without warning. Further, the barber was not involved in hiring or supervising the contractor and would not be responsible for the contractor's negligence. Hence, because no facts point to negligence by the barber, the customer is not likely to prevail

What is highly relevant for the defendant in defending a strict products liability claim?

The defendant manufacturer has a defense if the retailer discovered the defect during the course of an inspection but failed to warn the buyer. The same concepts of proximate cause governing general negligence and strict liability actions are applicable to strict liability actions for defective products. As with products liability cases based on negligence, the negligent failure of an intermediary to discover the defect does not cut off the supplier's strict liability. But when the intermediary's conduct becomes something more than ordinary foreseeable negligence, it becomes a superseding cause. The manufacturer can argue that the retailer's failure to take action after discovering a defect was not foreseeable and therefore cuts off the manufacturer's liability for the defect. The fact that the retailer could have discovered the defect during a reasonable inspection but failed to make any inspection would generally not be relevant to the manufacturer's defense; that typically would be considered ordinary foreseeable negligence which does not cut off the manufacturer's liability. Whether the retailer made a reasonable inspection of the product or that it was impossible to inspect the product are irrelevant to the retailer's defense. A retailer in a strict liability action may be liable for a manufacturing or design defect simply because it was a commercial supplier of a defective prod

A law enforcement officer was transporting a prisoner on a plane to testify in a criminal case. Unknown to those on the plane, an assassin hired to kill the prisoner had bribed an airport baggage handler to sneak a timed-release crate of poisonous snakes into the cargo hold of the plane. Once the crate was triggered to open, the snakes were able to slither into the passenger compartment through gaps in the conduits between the cargo hold and the passenger compartment. In the ensuing panic caused by the snakes, the officer was struck in the head by a fire extinguisher that another passenger threw at a snake, and suffered a severe concussion. The officer filed suit against numerous parties, including the person who designed the conduit system on that type of plane. At trial, evidence established that the design for the conduit system that he used had been rejected in the industry because of the danger of pressure loss between the cargo hold and the passenger compartment. An industry-approved design that the designer could have used would have kept the snakes from getting into the passenger compartment of the plane. As between the officer and the designer, which party is likely to prevail?

The designer will prevail because the assassin's actions were an unforeseeable intervening force. To establish a prima facie case for negligence, the following elements must be proved: (i) the existence of a duty on the part of the defendant to conform to a specific standard of conduct for the protection of the plaintiff against an unreasonable risk of injury; (ii) breach of that duty by the defendant; (iii) the breach of the duty by the defendant was the actual and proximate cause of the plaintiff's injury; and (iv) damage to the plaintiff's person or property. The general rule of proximate cause is that the defendant is liable for all harmful results that are the normal incidents of, and within the increased risk caused by, his acts. An indirect cause case is one where the facts indicate that a force came into motion after the time of defendant's negligent act and combined with the negligent act to cause injury to the plaintiff. Whether an intervening force will cut off the defendant's liability for the plaintiff's injury and be deemed superseding is determined by foreseeability. Here, the designer, as a professional designing a component of a plane, owed a duty of care to passengers such as the officer. He breached that duty of care by using a design for the conduit system that had been rejected in the industry because of the danger of pressure loss. His breach was the actual cause of the officer's harm because, but for his use of that design, the snakes would not have gotten into the passenger compartment of the plane. However, the conduct of the assassin in causing snakes to be placed on the plane is an unforeseeable intervening force. While criminal acts of third persons may be foreseeable if the defendant's negligence increased the likelihood of the crime being committed, there is nothing to suggest that the designer's negligence had any influence on the assassin's conduct. Hence, that conduct cuts off the designer's liability to the officer for the negligent design of the conduit system. (A) is incorrect. To establish strict tort liability, the plaintiff must prove that the defendant is a commercial supplier of a product. The designer, however, provided a service of designing a conduit system in a plane; because the facts do not suggest that he is a commercial supplier of a product, he cannot be held strictly liable. (B) is incorrect. Even assuming that the designer would be held to the high degree of care that common carriers owe their passengers, the officer must still establish the other elements of the tort. As discussed above, he would not be able to establish proximate cause under these facts. (D) is incorrect because the response by the other passenger is a foreseeable "reaction" force that does not cut off the causal connection between the act and the harm. If the designer were deemed to be a proximate cause of the snakes getting into the passenger compartment, the fact that the officer's injury was caused by the reaction of another passenger rather than a snakebite would not matter.

A homeowner purchased a ladder from a home supply retailer. While he was using the ladder, an improperly installed bolt fastening one of the rungs gave way, causing him to fall and break his leg. The homeowner sued the manufacturer of the ladder to recover damages for his injury. If it is established at trial that the home supply retailer could have discovered the defectively installed bolt if it had conducted a reasonable inspection of the ladder, what is the effect of the retailer's failure to inspect?

The failure of the home supply retailer to inspect the ladder has no legal effect on the manufacturer's liability, regardless of whether the plaintiff is suing in negligence or strict liability. Under either theory, an intermediary's negligent failure to discover a defect is not a superseding cause, so the defendant who supplied the defective product will still be liable. Thus, even if the home supply retailer were negligent in not discovering the defect, it would not relieve the manufacturer of liability.

A bolt of lightning struck a tree, causing it to fall on a farmer's fence which enclosed a pasture containing the farmer's large bull. The bull escaped through the broken fence and entered the neighbor's property. It gored a hiker who was crossing the neighbor's property without permission. In the hiker's action against the farmer based on strict liability, is the hiker likely to prevail?

The hiker will not prevail because strict liability does not apply to a bull, which is a domestic animal. The owner of a domestic animal, including a farm animal, is not strictly liable for injuries it causes, as long as the owner has no knowledge that the animal has abnormally dangerous propensities (i.e., propensities more dangerous than normal for that species). A bull is a domestic animal, and nothing in the facts suggests that the bull was more dangerous than normal for that type of animal. Hence, strict liability will not apply. (A) is incorrect because the rule for trespassing animals does not apply. The owner of a trespassing animal is strictly liable for harm done by the trespass as long as it was reasonably foreseeable. Here, the bolt of lightning caused the fence to break and allowed the bull to escape. This unforeseeable intervening force was the cause of the trespass; hence, the strict liability rule for trespassing animals does not apply here. (B) is incorrect because, as discussed above, strict liability does not apply for domestic animals with normal dangerous propensities. Only domestic animals with propensities more dangerous than normal for the species may subject the owner to strict liability. (D) is incorrect because the hiker's status as a trespasser on the neighbor's land is irrelevant as to the farmer's liability. If the hiker were a trespasser on the farmer's land, strict liability would not apply even if the bull were abnormally dangerous, but the farmer's liability is not affected by the hiker's status as to the neighbor. Note that if strict liability applied for harm from an animal trespassing on a neighbor's property, the hiker's status as a trespasser might be relevant because strict liability applies only to injured persons who were rightfully on the property. However, as discussed above in (A), that liability is inapplicable here because the bull's trespass was unforeseeable.

A housecleaning agency was given a key to a customer's house so that the agency could have its employees clean while the homeowner was away. After a maid sent by the agency had finished and left the homeowner's house, she went back because she had forgotten her cigarettes. She neglected to lock the door when she left the second time because she was already late for the next job. When the homeowner returned after a few days away, she discovered that her house had been ransacked and several items of jewelry stolen. The front door was open, and there were no signs of forced entry. If the homeowner brings an action against the agency that employed the maid, what is the likely result?

The homeowner will prevail because the maid's negligence increased the risk of criminal conduct by a third party. Criminal acts and intentional torts of third persons are foreseeable independent intervening forces if the defendant's negligence created a foreseeable risk that they would occur. Here, the maid's failure to lock the door was negligent because it created a risk of burglary; hence, the burglary does not cut off the agency's liability for the maid's negligence. As the maid's employer, the agency is vicariously liable under respondeat superior. (A) is wrong because there is nothing in the facts to indicate that the homeowner waived her right to bring tort claims against the agency; having a contractual relationship with a party does not automatically preclude bringing a tort action against the party. (B) is wrong because the burglary was not a superseding cause of the loss; it was within the increased risk caused by the maid's negligence. (D) is wrong because she reentered to retrieve a personal item that she had brought with her when she went to the job; her return just to get the item was within the scope of her employment and would not make her a trespasser.

What is standard for strict liability claim against manufacturer?

The motorist's claim against the car manufacturer, as a commercial supplier of the product, likely would be based on strict liability in tort. As such, he would only need to establish that the car was in a defective condition unreasonably dangerous to users in order to recover. A defective door latch would be a dangerous defec

Two neighbors who worked in a large city nearby alternated days driving. Because the commute took them through a crime-ridden area, one commuter was vigilant about keeping her car well-maintained, but the other failed to maintain her car or bring it in for servicing, despite the first commuter's complaints and dashboard warning lights indicating that it needed servicing. One evening after dark when the latter was driving them both home from work, her car died just as they were passing through a dangerous neighborhood. The passenger, who was calling for assistance on her cell phone, protested when the driver opened her door to look at the engine. Two assailants appeared and beat and robbed the driver and passenger. Does the passenger have a valid claim against the driver for her injuries?

The passenger has a valid claim against the driver because the jury could reasonably conclude that she was negligent. The driver owes a duty of ordinary care to his passenger regardless of whether that passenger is paying or not paying. The driver also owed a duty to act as a reasonable person would under emergency circumstances after the car was stopped. Her opening of the car door, as well as her failure to maintain her car, could be found to be negligent under the circumstances. The acts of the criminals were foreseeable, because every day the passenger and the driver commuted through this dangerous neighborhood. (A) is incorrect because this is the standard of care owed to a paying passenger. (C) is incorrect because whether criminal acts of third persons are superseding depends on foreseeability. Here, the criminal activity was foreseeable under the circumstances and therefore does not constitute a superseding cause. (D) is incorrect. The driver owed a duty to her passenger to use reasonable care while in this dangerous area.

A passenger suffered a broken arm from an automobile accident caused by his driver's negligence in running through a red light. The passenger was taken by ambulance to a nearby hospital for treatment. There, the emergency room physician negligently reset the bone in the passenger's arm. As a result, the passenger never recovered full use of his arm and his earnings as a carpenter were permanently reduced. The jurisdiction retains traditional contribution rules based on equal shares in cases applying joint and several liability. If the passenger brings suit against the driver for the damage to his arm, the passenger will recover what amount? Why?

The passenger will recover all of his damages from the driver. Because the damage resulting from medical malpractice was a foreseeable result of the driver's negligence, the driver is liable for such damage even though the negligent doctor contributed to the passenger's damages; i.e., the doctor's negligence is not a superseding cause and therefore does not cut off the driver's liability for the passenger's injuries. (B) is wrong because joint and several liability allows the plaintiff to recover all of his damages from one of the joint tortfeasors. (C) is wrong because contribution rules apply only to a defendant who has paid the full judgment to the plaintiff and is seeking contribution from another tortfeasor; they have no relevance to the plaintiff's recovery against the defendant. (D) is wrong because the passenger could maintain an action for all his damages directly against the driver with no need to join the doctor.

On the way home from a nightclub, a passenger began yelling at the designated driver claiming that he was not taking the best route back to her house. The driver disagreed and contended that his route was the quickest. The passenger impulsively grabbed the steering wheel, causing the car to swerve and strike a pedestrian, injuring him. At trial, the pedestrian established that the driver's license had expired the day before the accident. The driver's traffic record qualified him for an automatic renewal of his driver's license, but he had forgotten to submit it in time. A statute in the jurisdiction makes it an offense to drive a vehicle on any public road in the state without a valid driver's license. Will the pedestrian prevail?

The pedestrian will not prevail because the passenger's grabbing of the wheel is the negligent conduct that caused the pedestrian's injuries. To establish a prima facie case for negligence, the following elements must be proved: (i) the existence of a duty on the part of the defendant to conform to a specific standard of conduct for the protection of the plaintiff against an unreasonable risk of injury; (ii) breach of that duty by the defendant; (iii) the breach of the duty by defendant was the actual and proximate cause of plaintiff's injury; and (iv) damage to plaintiff's person or property. Here, the driver's actions were an actual cause of the pedestrian's injury because, but for the driver's driving and the passenger's grabbing the steering wheel, the injury to the pedestrian would not have happened. However, the driver's actions were not a proximate cause of the injury because the passenger's grabbing of the steering wheel was a superseding intervening force. A superseding force is one that serves to break the causal connection between the initial wrongful act and the ultimate injury, and itself becomes a direct immediate cause of such injury. Thus, the first actor would be relieved of liability from the consequences of his antecedent conduct. The passenger's conduct in suddenly grabbing the steering wheel was an unforeseeable intervening force creating an unforeseeable harmful result, and thus constituted a superseding force. Consequently, the driver would be relieved of any negligence liability since the passenger's actions were the proximate cause of the accident.

What does a plaintiff need to do to prevail for a strict liability claims for abnormally dangerous activity?

To prevail, a plaintiff need only show that a reasonable person could have foreseen the risk of harm, regardless of whether the defendant did not. If the plaintiff was not foreseeable, the strict liability claim is not established. The defendant's liability for an abnormally dangerous activity extends only to foreseeable plaintiffs, who are persons to whom a reasonable person would have foreseen a risk of harm under the circumstances. Note, though, that the nature of the abnormally dangerous activity may create a large class of foreseeable plaintiffs. If the type of harm was not foreseeable, the plaintiff cannot establish a strict liability claim. The harm must result from the kind of danger to be anticipated from the abnormally dangerous activity; i.e., it must flow from the "normally dangerous propensity" of the condition or thing involved. An unforeseeable intervening force may allow a defendant to avoid liability in a strict liability action for an abnormally dangerous activity. The same rules govern causation for strict liability as they do for negligence, and thus a defendant's liability may be cut off by an unforeseeable intervening force that brings about the injury.

A man purchased a new power boat with an inboard engine from a boating supply store. The boating supply store properly inspected the boat before delivery, but did not detect a virtually invisible manufacturing defect in the boat's steering mechanism. Later that summer, the man was entertaining some friends on his boat on a lake near a dam. There were some warning pylons near the dam, warning boaters to stay clear. The man decided to show off for his friends by weaving his boat in and out of the warning pylons. As he rounded the last of them, the steering mechanism of his boat jammed, and the boat crashed into the dam. The man was severely injured. The man brings an action for damages against the boating supply store on a theory of strict liability in tort in a jurisdiction that does not apply its comparative fault rules to strict liability actions. Who will prevail?

To recover on a theory of strict tort liability, the man must show that his injuries were caused by an unreasonably dangerous defect in the boat that existed when the boat left the boating supply store's control; (C) is the only alternative that reflects this requirement. A prima facie case in products liability based on strict tort liability consists of the following: (i) the defendant is a commercial supplier; (ii) the defendant produced or sold a defective product; (iii) the product was the actual and proximate cause of the plaintiff's injury; and (iv) the plaintiff suffered damages to person or property. Examples of commercial suppliers include manufacturers, retailers, wholesalers, and assemblers. The second element is established by proving that the product is in a defective condition unreasonably dangerous to users. A plaintiff need not prove that the defendant was at fault in selling or producing a dangerous product. To prove actual cause, a plaintiff must trace the harm suffered to a defect in the product that existed when the product left the defendant's control. Here, because the steering failed due to a defect present when the boat left the manufacturer, that defect must also have been present when the man bought the boat from the boating supply store, the retailer. This defect rendered the boat unreasonably dangerous to users such as the man. The boating supply store sold the boat in this defective condition, and the defect actually and proximately caused the man to incur severe personal injuries. Thus, (C) states why the man will prevail. (A) is incorrect because the inspection of the boat by the boating supply store prior to sale would be relevant to a negligence action, but not to one based on strict liability. Strict liability will still lie because the boat left the boating supply store's control with a defect that rendered it unreasonably dangerous. (B) is incorrect because ordinary contributory negligence is not a defense in strict liability actions in jurisdictions that do not apply comparative fault rules in these cases. To the extent that the man is "misusing" the boat by weaving in and out of the pylons, it is a reasonably foreseeable misuse that the commercial supplier must take into account. To avail itself of the man's conduct as a defense, the boating supply store must show that the man voluntarily and unreasonably encountered a known risk. The facts herein do not indicate any such knowing assumption by the man of the risk of harm from the defective steering mechanism. (D) is incorrect because it does not establish the causation element. The boating supply store's strict tort liability depends on whether the steering mechanism failed because of a defect present at the time it sold the boat to the man. If the boat was not defective at the time of sale, or if any defect that was present had nothing to do with the failure of the steering mechanism, the boating supply store will not be liable for a subsequent failure of the steering mechanism from some other cause.

T or F - The plaintiff may apprehend an immediate harmful or offensive contact without feeling fear or intimidation

True

Depending on the circumstances, strict liability may be imposed on the owners of what type of animals?

Wild animals, domestic animals, and trespassing animals Depending on the circumstances, strict liability may be imposed on the owners of wild animals, domestic animals, and trespassing animals. Unless an owner of wild animals can rely on a public duty exception (e.g., a zookeeper), the owner is strictly liable for injuries caused by the wild animals, even those kept as pets. An owner is strictly liable for the damage done by the trespass of his animals (other than household pets) as long as the damage was reasonably foreseeable. It does not matter that the owner acted with reasonable care to keep them from trespassing. Normally, the owner of a domestic animal is not strictly liable for injuries it causes. Strict liability will attach, however, if the owner knows of the domestic animal's dangerous propensities (i.e., propensities more dangerous than normal for that species). This rule applies even if the animal has never injured anyone.

A shopper was in a large department store that was remodeling its menswear department and had hired a contractor to do the work. A carpenter employed by the contractor was working on the remodeling job. When the carpenter left the store to take a break, she left a carpenter's level projecting out into one of the aisles, unbeknownst to any store employees. Shortly before she returned 15 minutes later, the shopper came down that aisle and tripped over the level. He fell and struck his head on the sharp corner of a display case. The shopper required hospitalization and sued the store for his injuries. Will the shopper prevail in his suit against the store?

Yes. The shopper will prevail because the employee of the contractor hired by the store left the level in the aisle. The general rule that a principal will not be vicariously liable for the acts of its independent contractor's agent is subject to several broad exceptions, including one for duties that are nondelegable because of public policy considerations. One of these duties is the duty of a business to keep its premises safe for customers. Hence, a business would be liable for the negligence of an employee of an independent contractor to the same extent as for the negligence of its own employee. Here, the carpenter was employed by the contractor, which was hired by the store. The carpenter breached the duty owed to customers such as the shopper by leaving the level projecting out into one of the aisles. The shopper was injured as a result, so he will prevail in a suit against the store. (B) is wrong. As part of the duty owed to customers, the store employees have a duty to make reasonable inspections of their premises to discover unsafe conditions (such as if a customer had spilled something slippery in an aisle). However, regardless of whether they had a reasonable time to discover the level, the store is liable because it is responsible for the carpenter's conduct. (C) is wrong because, as discussed above, the store is liable under these circumstances for the conduct of its independent contractor's employee. (D) is wrong because the store is liable regardless of the knowledge of its employees.

Which of the following best describes res ipsa loquitur?

es ipsa loquitur deals with those situations where the fact that a particular injury occurred may itself tend to establish the breach of a duty owed, because the type of injury that occurred would not normally occur in the absence of negligence. Where the facts are such as to strongly indicate that the plaintiff's injuries resulted from the defendant's negligence, the trier of fact may be permitted to infer the defendant's liability. Res ipsa loquitur does not refer to custom or usage establishing the standard of care. Custom or usage may be introduced to establish the standard of care in a given case. Unlike res ipsa loquitur, however, customary methods of conduct do not furnish a test that conclusively controls the question of whether certain conduct amounted to negligence. Res ipsa loquitur also does not refer to the defendant's violation of a statute as establishing the existence of a duty and breach thereof. Proof that a defendant violated an applicable statute may establish the existence of a duty owed to a plaintiff and a breach thereof, but that is not an application of circumstantial evidence to show breach of duty, as res ipsa loquitur is. Res ipsa loquitur does not establish breach of duty as a matter of law. It is still up to the trier of fact to accept or reject the evidence.


Related study sets

Uworld Adult Gastrointestinal and Cardiovascular

View Set

L'engagement: Les Mains Sales- Sartre, Quotations, Pre-U French, Paper 4: Topics and Texts

View Set

PSYCH 25 Ch 5: Physical Development

View Set

GRE Math Things that I Forget too Easily

View Set

Gulliver's Travels by Jonathon Swift

View Set

Vocabulary Workshop Level F Unit 2 Synonyms

View Set

454 Investments McGraw-Hill Q&A Ch-Q#-letter

View Set

How Blood Travels Through the Body

View Set